Vous êtes sur la page 1sur 103

Institute for All Competitive Exams

Analytical Reasoning
Passage and Conclusion.................................................................................................................... 2
Practice Questions: Passage and Conclusion ............................................................................... 4
Descriptive Answers (Passage and Conclusion) ........................................................................... 7
Statement and Arguments .............................................................................................................. 10
Practice Questions: Statement and Arguments .......................................................................... 12
Descriptive Answers (Statement and Arguments) ..................................................................... 16
Statement and Assumptions........................................................................................................... 18
Practice Questions: Statement and Assumptions I .................................................................... 21
Descriptive Answers (Statement and Assumptions I) ............................................................... 26
Practice Questions: Statement and Assumptions II ................................................................... 28
Descriptive Answers (Statement and Assumptions II) .............................................................. 37
Practice Questions: Statement and Assumptions III ................................................................. 40
Answer key (Statement and Assumptions III) ........................................................................... 43
Statement and Conclusions ............................................................................................................ 44
Practice Questions: Statement and Conclusions I ..................................................................... 46
Descriptive Answers (Statement and Conclusions I) ................................................................. 51
Practice Questions: Statement and Conclusions II .................................................................... 53
Answer Key: Statement and Conclusions II ............................................................................... 56
Statement and Course of Action ..................................................................................................... 57
Practice Questions: Statement and Course of Action ................................................................. 59
Descriptive Answers (Statement and Course of Action) ............................................................ 67
Practice Questions: Statement and Course of Action II ............................................................. 70
Answer Key (Statement and Course of Action II) ...................................................................... 72
Argument Strengthen/ Weaken ..................................................................................................... 73
Answer Key (Argument Strengthen / Weaken) .......................................................................... 75
Probably True & False .................................................................................................................... 76
Answer Key (Probably True & False) .......................................................................................... 77
Conditional Selection ...................................................................................................................... 78
Answer Key (Conditional Selection) ........................................................................................... 82
Cause Effect Labelling I .................................................................................................................. 83
Answer Key (Cause Effect Labelling I) ....................................................................................... 84
Cause Effect Labelling II................................................................................................................. 85
Answer Key (Cause Effect Labelling II) ...................................................................................... 86
Inequalities ..................................................................................................................................... 87
Answer Key (Inequalities)........................................................................................................... 90

Analytical Reasoning #3094, Sector 37D, Chandigarh. E-Mail: info@gjtutorial.com Page 1


Institute for All Competitive Exams
Passage and Conclusion

In this type of questions, a passage is given on any topic followed by conclusions in the form of
statements. A candidate is required to mark the answer in the following way
(a) If the conclusion is definitely true.
(b) If the conclusion is probably true, though not definitely true.
(c) If the conclusion is neither true nor false because the data given in the passage are inadequate.
(d) If the conclusion is probably false, though not definitely false.
(e) If the conclusion is definitely false.

Note: A student must note that all these conclusions are to be arrived at in the light of what has
been discussed in the passage.

Example:

Passage:
―Now the question of the freedom of the press becomes an important question in view of the fact
that the press has vast potentialities for both good and evil. The press is as much a passion rouser
as it is a peace maker. In the days of discords and dissensions, it bars the door to reconciliation or
at least the quarrel is prolonged beyond its limits. Knowledge is not always good and sometimes
ignorance is bliss.‖

Option (a): Definitely true:


If the conclusion directly follows the statement or group of statements given in the passage, we can
arrive at option (a) because the conclusion is definitely true. It is important to note that in such
cases the conclusion is the direct result of the statement and no indirect inference is derived from
the given statement. Let us analyse the conclusions which are definitely true in the light of this
rule.
Conclusion 1
Press has multi-dimensional potentialities.
Analysis:
It is very clearly mentioned in the passage that press possesses vast potentialities and can play
different roles during work and peace time.
Conclusion 2
Press can rouse the passions of the public.
Analysis:
The author has described the press as a passion rouser as well as peace maker.
Since both the conclusion drawn from the passage can directly be derived from the passage, hence
both the definitely true. We have not relied on any indirect inference.
Option (b): Probably true
The word 'probable' means that there are chances of occurring or not occurring of an event. This
option is available for testing a conclusion when it is not definitely true. In other words, it does not
follow the statement directly. It is to be noted here that a conclusion is not definitely true because
there exists little chance for its being wrong as well.
Therefore, there, is every possibility of its swing towards truthfulness.
Conclusion 3
It is a folly to be wise where ignorance is bliss.
Analysis:
In the last line of the paragraph, it is mentioned that knowledge is not always good and sometimes
ignorance is bliss. It is to be noted that we cannot infer anything definite from this line because the
words 'always' and 'sometimes' reduce the chance of the conclusion being definitely true or false.

Analytical Reasoning #3094, Sector 37D, Chandigarh. E-Mail: info@gjtutorial.com Page 2


Institute for All Competitive Exams
In the light of the rule under consideration, the conclusion derived has greater swing towards
probability of being true than being false. Hence the conclusion is probably true.
Option (c): Data inadequate
In such option the candidates do not have relevant information to arrive at the conclusion. In other
words, data contained in the passage are inadequate. As such the option (c) - data inadequate, is
required to be opted for by the candidate.
Conclusion 4:
―Salary of journalists should be increased to suppress their resentment.‖
Analysis:
This conclusion speaks of salary of journalists. Nothing about salary of journalists has been
discussed in the passage. Additional data, therefore, are required to arrive a Conclusion. Hence, we
opt for option (c)-data inadequate.

Option (d): Probably false


The criterion to opt for this option is similar to the one applied in the case of option (b) - Probably
true. Whenever the conclusion is not definitely false, it is so because there exists little chance of its
being true. In such cases, the conclusion is said to be probably false. Here the swing seems to be
towards falsity and the conclusion is termed probably false.
Conclusion 5
Knowledge is good.
Analysis:
Though the conclusion contradicts the statement of the passage that knowledge is not always good,
yet the conclusion cannot be taken for as definitely false. All the same, the use of word 'always' in
the statement reduces the chance of the conclusion being definitely true and there is an apparent
swing towards falsity. Hence, the option (d) - probably false is correct.

Option (e): Definitely false


The student arrives at this option when direct definite information is available to prove that the
conclusion is totally contrary to the information given in the passage.
Conclusion 6
The question about the freedom of press is not a significant one.
Analysis:
Now, if we analyse the conclusion in the light of the information available in the passage, it will be
observed that the conclusion contradicts the first line in the passage which reads ―The question of
freedom of press becomes an important question‖. Therefore, the conclusion is definitely false.

Analytical Reasoning #3094, Sector 37D, Chandigarh. E-Mail: info@gjtutorial.com Page 3


Institute for All Competitive Exams
Practice Questions: Passage and Conclusion

Passage (I)
In the absence of an integrated sugar field to sale policy, the Indian sugar industry has become
victim of surplus production and price mismatch of sugarcane and finished sugar. Despite a lower
estimated sugar production at around 12.8 million tonnes for 1996-97 against 16.7 million tonnes
in the previous year, the total availability is put to 20.8 million tonnes including a carryover stock
of 8 million tonnes from the previous year. Of this the domestic consumption may not exceed 13.5
million tonnes. Though the industry could export 10.5 million tonnes to different countries during
1995-96, this year's export policy, existing norms and international market conditions may bring
down the export quantity to half a million tonnes.

1. There may be a fall in the sugar price during 1997-98.


2. The quantum of current year's sugar production is close to the envisaged.
3. India's sugar export was the highest in recent times during 1995-96.
4. India need not import sugar during the next few years.
5. India's export policy has made the sugar price non-competitive in the international market.

Passage (II):
Urban service has not expanded fast enough to cope with urban expansion. Low investment
allocations have tended to be under spent. Both public (e.g. water and sewage) and private (e.g.,
low income area housing) infrastructure quality has declined. The impact of the environment in
which children live and the supporting services available to them when they fall ill, seems clear.
The decline in average food availability and the rise in absolute poverty, point in the same
unsatisfactory directions.

6. There is nothing to boast about urban services.


7. The public transport system is in the hands of private sector.
8. Birth rate is higher in urban areas as compared to rural areas.
9. Low cost urban housing is on the priorities.
10. The environment around plays an important role on the health status.
11. Though adequate provisions of funds were made but they were received under spent.

Passage (III):
There is some controversy about the percentage of population below the poverty line in India. The
criterion for the poverty line is based on person's nutritional requirement in terms of calories. It is
assumed that the minimum nutritional requirement per person per day in rural areas is 2400
calories whereas it is 2200 calories in urban areas. If the household is unable to bear the
expenditure for this level of nutrition, it is categorised as below the poverty line. There is also a
view that, along with calories, the amount of protein intake be treated as a criterion as it is related
to physical energy, mental alertness and resistance to infection.

12. Many Indians, who are below the poverty line, get necessary amount of proteins.
13. People living above the poverty line are less likely to suffer from infections.
14. India's poverty alleviation programmes can only succeed after reaching agreement about the
poverty line.
15. People in urban areas are less physically weak compared to people in rural areas.
16. In other countries, there is no controversy about defining the poverty line.

Passage (IV)
A radical new surgery procedure launched at, not long ago in India, is holding out fresh hope for
patients of cardiac myopathy, or enlargement of the heart. The technique now in India, allows
Analytical Reasoning #3094, Sector 37D, Chandigarh. E-Mail: info@gjtutorial.com Page 4
Institute for All Competitive Exams
patients to go home two weeks after the operation to lead a near-normal sedentary life. Cardiac
myopathy is a condition that has a variety of causative factors. An attack from one of the 20
identified viruses, parasite infection, long-term alcohol abuse and blood pressure could bring it on,
and in rare cases, and it could follow child birth and is even known to run in families. The
condition is marked by an increase in the size of heart‘s chambers and a decrease in the efficiency
of pumping.

17. The cardiac myopathy slows down the heart beat.


18. Cardiac myopathy is hereditary.
19. Earlier the patients suffering from cardiac myopathy were required to travel abroad for such
operation.
20. The new technique was never tried in India in the past.
21. The efficiency of the heart is inversely proportional to the size of the heart.

Passage (V)
―Indian rice exporters might face tough competition in the international market‖, ―allege
exporters. International demand for Indian rice surfaced, either due to its low prices or due to the
crop failure in other competitive countries. However, our competitors are having a good crop of
rice this year and which indeed sounds as a threat for India, they pointed out. The leading
producing countries like Thailand, Vietnam, Myanmar, USA and Pakistan are bringing down their
prices at par with Indian prices. Internationally, these counties have reduced their prices by Rs. 50
per ton in lower grades, coming almost at par with Indian prices. Indian rice currently quoting
between $ 260 and $ 265 per ton. However, in high grade the difference still remains high.
Therefore, India can expect to be competitive in high grades provided the quality and deliveries are
ensured in time. Pakistan is our main competitor and it has reduced its price by 15 per cent coming
almost in line with the Indian prices. The very fact that international producers have started to
reduce their prices is a signal for Indian exporters to gear up for 1996-97.

22. Indian has no potential threat in the international market for exporting of rice.
23. Quality and delivery of goods in time are the two main components for being in the forefront.
24. To remain in the international market, India should reduce its price.
25. India should increase production of rice to gear up for 1996-97.
26. Indian crops were inferior in quality than the leading rice producing countries.

Passage (VI)
The most empirical argument in favour of prayer is that it relieves the mind of tension, which is the
natural concomitant of a life of hurry and worry. It neutralises mental repressions and purifies the
sub consciousness. It releases an extra amount of hope and energy and thus enables a person to
face life squarely. Life is not all sweetness, enjoyment and success; it is drudgery, pain and failure
too.

27. Prayer is an important part of all religions.


28. People are interested in finding out effects of prayer.
29. Sub consciousness plays no role in our lives.
30. There are some arguments for the benefits of prayer, which can be tested.
31. Prayer, by direct miracles, removes hurdles and difficulties from our lives.

Passage (VII)
According to trade estimates, the black pepper crop will be in the range of 75,000 to 90,000
tonnes during the pepper season from November 1989 to October 1990. The crop will thus be the
double of that for 1988-89 season. Besides good monsoon, the record production is attributed to
the crash programme launched in Kerala by the State government last June. Although 80 percent

Analytical Reasoning #3094, Sector 37D, Chandigarh. E-Mail: info@gjtutorial.com Page 5


Institute for All Competitive Exams
of the country's total black pepper production is grown in Kerala, the yield per hectare is low at
300 kg as compared to 550 kg in Indonesia and 1,200 kg in Brazil.
32. India exports black pepper more than Brazil.
33. The black pepper crop during 1988-89 was about 30,000 tonnes.
34. In no other State in the country, black pepper yield is more than 300 kg per hectare.
35. Central government helped Kerala government in implementing the crash programme.
36. Good water supply is needed for cultivating black pepper.

Passage (VIII)
The only dark cloud in the economic horizon is the rather unsatisfactory foreign exchange and one
may hope that the industry will play its role in improving it by contributing to its export. The all
round progress of our company is continuing. I am happy to report that during the first half of the
current year (April-September 1989), there has been further improvement in its working.

37. The country is having inadequate foreign exchange reserves.


38. The current financial year of the company will end on 31st March 1990.
39. In foreign countries, there is no demand for the product manufactured by our company.
40. The government of the country has asked the company to export its goods.
41. The report is presented by the chairman of the company to the share holders.

Passage (IX)
Of the roughly 4,40,000 children, who currently languish in America's foster-care system, 20,000
are available for adoption, most of them are older children between the ages of 6 and 12. Among
the adoptable children, 44% are white and 43% are black. But 67% of all families waiting to adopt
are white, and many of them are eager to take a black child. The hurdles, however, are often
formidable. Though only three US States - Arkansas, California and Minnesota - have laws
promoting race matching in adoptions, 40 other favour the practice.

42. Children beyond 12 years of age are less suitable for adoption.
43. White children are being preferred for adoption by majority of black families.
44. Majority of the US States have laws that discourage the practice of adoption.
45. Among the adoptable children, at least 10 percent are neither black nor white.
46. Two-third of the white families are waiting to adopt a black child.

Passage (X):
Data available from the National Institute of Nutrition (NIN) compares, separately for boys and
girls, the average height recorded during 1979 with the average for the period 1974-79 at different
ages in the 10 States covered by the NNMB survey. The 1979 height turns out to be neither
uniformly higher nor uniformly lowers than the 1974-79 average height at ages 2, 5 and 9 in most
of the States. However, the comparison at age 13 is more meaningful as it represents the
cumulative result of childhood growth. At age 13, the average height recorded for boys in 1979 is
lower than the average for 1974-79 in only one State, Karanataka. For girls at this age, the average
height recorded in 1979 turns out to be lower than the 1974-79 average in four States-Andhra
Pradesh, Gujarat, Madhya Pradesh and Uttar Pradesh. In all other states, the 1979 average was
greater than or equal to the 1974-79 average.

47. There are only two institutes - NIN and NNMB which have collected data on average height.
48. The data from Karnataka was collected by both NIN and NNMB.
49. In Karnataka, the average age recorded in 1979 for both boys and girls is lower than that for
1974-79.
50. More States indicated higher average in 1979 as compared to that of 1974-79.
51. Both the institutes NIN and NNMB collected data from 10 States only.
52. Separate statistics for boys and girls help understand the data in a better way.
Analytical Reasoning #3094, Sector 37D, Chandigarh. E-Mail: info@gjtutorial.com Page 6
Institute for All Competitive Exams
53. There may not be large differences in the height of boys and girls from 2nd to 5th year.

Descriptive Answers (Passage and Conclusion)

1 (a):
Conclusion speaks of probability of the fall in price for the year 1997-98. It is given in the passage
that there has been surplus production in the year
1996-97. Hence conclusion is definitely true.
2 (e):
Conclusion is definitely false because it is clearly mentioned that production of sugar is 20.8
million tonnes against an estimate of 12.8 million tonnes.
3 (c):
No data is given regarding the comparative sugar export during 1995-96.
4 (b):
Since there has been surplus production of sugar, hence, it is likely that India will not require to
import sugar during next few years.
5 (c):
Data given in the passage are not adequate to say anything about the conclusion.
6 (a):
It is clearly mentioned in the opening lines of the passage that urban services have deteriorated
due to meagre finance and related support.
7 (c):
It is not mentioned anywhere in the passage that public transport system is in the hands of private
sector.
8 (c):
Data regarding comparative birth rate are not given in the passage. Hence data are inadequate.
9 (b):
It is given in the passage that private infrastructure quality, e.g., low income area housing has
declined. Therefore, it is very likely that low cost urban housing gets priority.
10 (a):
This is clearly given in the passage that environment plays an important role on the health status.
Hence, conclusion definitely follows from the passage.
11 (e):
Conclusion is definitely false as it is given in the second line of passage that low investment
allocations have tended to be under spent.
12 (c):
Data given in the passage are inadequate.
13 (b):
It is given in the last line of the passage that protein is related to resistance to infections. Since
people above poverty line may get sufficient amount of protein intake, and so are likely to be less
sufferer from infections.
14 (c):
Nothing has been discussed in the passage regarding agreement about poverty line.
15 (b):
It is probably true because calorie requirement per person in urban area is less than that of rural
area.
16 (c):
Nothing has been given in the passage regarding the poverty line in the ether countries.
17 (b):
Conclusion is probably true because it is given in the last line that cardiac myopathy is a result of
enlargement of heart's chamber and decrease in the efficiency of pumping.

Analytical Reasoning #3094, Sector 37D, Chandigarh. E-Mail: info@gjtutorial.com Page 7


Institute for All Competitive Exams
18 (a):
Conclusion is definitely true as it is given in the passage that cardiac myopathy could follow child
birth.
19 (c):
Data given in the passage are inadequate to analyse this conclusion as nothing has been given
regarding the facility available for this disease earlier in India. 20 (a):
Opening line of the passage reads ―A radical new surgery procedure launched not long ago is
holding fresh hopes.‖ This implies that this surgery has not been used in India earlier.
21 (a):
Conclusion is definitely true from the last line of the passage that the condition is marked by an
increase in the size of heart's chamber and a decrease in efficiency of pumping.
22 (e):
Conclusion is definitely false as it has been mentioned in the paragraph that Indian rice exporters
are facing tough competition.
23 (a):
It has been given in the passage that India has to ensure quality and delivery on time to be
competitive in the international market.
24 (a):
Since other countries have reduced prices hence India should also reduce the price to be in
competition. Hence, conclusion is definitely true.
25 (a):
Conclusion is definitely true as to cope with the competition, either India has to increase
production or reduce the price.
26 (c):
Data given in the passage are inadequate as nothing has been given as to the quality of rice.
27 (c):
Data are not adequate as it has not been given anywhere in the passage whether prayer forms an
important part of all religions or not.
28 (e):
Conclusion is definitely false because effects of prayer are arguable as per opening line of the
passage.
29 (e):
Conclusion is definitely false because it has been given in the passage that prayer purifies the sub
consciousness. Hence, subconscious has a role in our lines.
30 (a):
Conclusion is definitely true as it has been given in the opening line of the paragraph.
31 (e):
Conclusion is definitely false because as per passage, prayer has an indirect, impact on the life of
human beings
32 (c):
Data given in the passage are inadequate because nothing has been discussed regarding the
comparative export figures between India and Brazil.
33 (e):
Conclusion is definitely false because the lowest estimated production in 1989-90 is 75,000
tonnes; double that of 1988-99. Hence, the production of 1988-89 will be more than 30,000
tonnes.
34 (b):
Conclusion is probably true because Kerala State has dominated in black pepper production.
35 (c):
Data are inadequate to say anything about central help to Kerala for the black pepper production.
36 (a):
It is given in the passage that good monsoon has attributed to the good production of the black
pepper, hence conclusion is definitely true.
Analytical Reasoning #3094, Sector 37D, Chandigarh. E-Mail: info@gjtutorial.com Page 8
Institute for All Competitive Exams
37 (a):
Conclusion is definitely true as it is clear from the opening line of the passage which reads ―The
only dark cloud in the economic horizon is the rather unsatisfactory foreign exchange.‖
38 (a):
Conclusion is definitely true as it is clear from the last line that first half of the current year is from
April to September.
39 (c):
Data are inadequate to say anything about the demand of Indian product in foreign countries.
40 (c):
Data are inadequate.
41 (c):
Data are inadequate as it is not given in the passage that chairman of the company presents its
report to the shareholders.
42 (b):
Conclusion is probably true because most of the adoptable children belong to the age group
between the ages of 6 and 12.
43 (c):
Data are not adequate in the passage.
44 (e):
Definitely false. All the States encourage adoption. Only three States promote adoption with
certain restriction.
45 (a):
Definitely true. Because total number of black and white children out of 100 = 44 + 33 = 87 and
remaining = 13%.
46 (b):
The statement is probably true, because of 100 = 66.66% which is near to 67%.
47 (c):
It is not given in the passage that there are only two institutes. There can also be more than two
institutes. So data are inadequate.
48 (a):
It is given in the passage that data from Karnataka were collected by both the institutes. So the
conclusion is definitely true.
49 (e):
The last line of the passage indicates that the height of girls is not lower in Karnataka.
50 (a):
It is very clear from the last line of the passage.
51 (c):
It is not given in the passage that institutes - NIN and NMMB collected data from 10 States only.
52 (b):
Conclusion seems to be probably true as it can be inferred from the passage that separate statistics
certainly help.
53 (b): Conclusion is probably true as it is clearly given in the passage that at the ages of 2, 5 and 9
comparative heights neither turn out to be uniformly higher nor lower.

Analytical Reasoning #3094, Sector 37D, Chandigarh. E-Mail: info@gjtutorial.com Page 9


Institute for All Competitive Exams
Statement and Arguments

In these questions a statement is followed by two arguments. Candidates are required to


distinguish between the strong and weak, argument. Generally, both the arguments are contrary to
each other to the positive and negative results of the action as mentioned in the statement. A
strong argument is that which touches the practical and real aspect of the situation as described in
the statement. A weak argument is very simple, superfluous, and long drawn one.

Following points should be taken into consideration while choosing a strong argument.
 A strong argument should give die realistic diagnosis of the situation described in the
statement.
 A strong argument should give the deep analysis of the topic dealt within the statement.
 A strong argument should relate with the statement and be supported up by facts or established
notions.
 A strong argument should not be mere reiteration of the situation given in the statement.

Solved Examples:

Directions: Study the instructions carefully and answer the questions that follow.
In making decisions about important question, it is desirable to be able to distinguish between
―strong‖ and ―weak‖ arguments so far as they relate to the questions. ―Weak‖ arguments may not
be directly related to question and may be of minor importance or may be related to the trivial
aspect of the question. Each question below is followed by two arguments numbered I and II. You
have to decide which of the arguments is a 'strong' argument and which is a weak' argument.
Give answer:
(a) If only argument I is strong
(b) If only argument II is strong
(c) If either I or II is strong
(d) If neither I nor II is strong and
(e) If both I and II are strong.

1. Statement:
Should school education be made free in India?
Arguments:
I. Yes, this is- the only way to improve the level of literacy,
II. No, it will add to the already heavy burden on the exchequer.
Solution:
Argument I is strong because word 'only' makes the argument weak as it is not the real and
practical solution to improve the level of literacy. Argument II is strong as it describes the practical
problem which may arise out of the decision of making education free in India; hence (b) is our
answer.

2. Statement:
Should there, be complete ban on manufacture of fire crackers in India?
Arguments:
I. No, this will render thousands of workers jobless.
II. Yes, the fire cracker manufacturers use child labour to a large extent.

Solution:
Both the arguments refer to the practical consequences of the action mentioned in the statement
and hence are strong. Therefore, (e) is the correct answer.

Analytical Reasoning #3094, Sector 37D, Chandigarh. E-Mail: info@gjtutorial.com Page 10


Institute for All Competitive Exams
3. Statement:
Should young entrepreneurs be encouraged?
Arguments:
I. Yes, they will help in the industrial development of the country.
II. Yes, they will reduce the burden, on the employment market.

Solution:
It is very clear that encouragement to the young entrepreneurs will open up the fields for setting
up of new industries. Hence, it will help in industrial development. Consequently, more job
opportunities will be created. Therefore, both the arguments are strong. Hence (e) is our answer.

4. Statement:
Should luxury hotels be banned in India?
Arguments:
I. Yes, these are places from where international criminals operate.
II. No, affluent foreign tourists will have no place to stay.

Solution:
The luxury hotels are symbol of country's standard and a place for staying the affluent foreign
tourists. So, argument II is a strong one. Argument I is a weak argument because ban on luxury
hotels is not a way to end the international criminals.

5. Statement:
Should election expenses to central and state legislature be met by the Government?
Arguments:
I. Yes, it will put an end to political corruption.
II. No, it is not good in any country.

Solution:
Argument I is strong one as this decision will help to eliminate political corruption. Argument II
does not give any logic, hence a weak argument. Therefore, (a) is our answer.

Analytical Reasoning #3094, Sector 37D, Chandigarh. E-Mail: info@gjtutorial.com Page 11


Institute for All Competitive Exams

Practice Questions: Statement and Arguments

1. Statement:
Should we impart sex education in schools?
Arguments:
I. Yes, all the progressive nations do so.
II. No, we cannot impart it in co-educational school.

2. Statement:
Should there be a ban on product advertisement?
Arguments:
I. No, it is an age of advertising. Unless your advertisement is better than that of your other
competitors, the products will not be sold.
II. Yes, the money spend on advertising is very huge and it inflates the cost of the product.

3. Statement:
Should there be a ceiling on the salary of top executives of multinationals in our country?
Arguments:
I. Yes, otherwise it would lead to unhealthy competition and comparison, and our own industry
would not be able to withstand that.
II. No, with the accent of liberalisation of economy, any such move would be counter-productive.
Once the economy picks up, this disparity will be reduced.

4. Statement:
Should the government levy tax on agricultural income also?
Arguments:
I. Yes, that is the only way to fill government's coffer.
II. No, eighty percent of our population live in rural areas.

5. Statement:
Should public holidays be declared on demise of important national leaders?
Arguments:
I. No, such unscheduled holidays hamper national progress.
II. Yes, people would like to pay their homage to the departed soul.

6. Statement:
Keeping in consideration the longevity of life in India, should the age limit for retirement in
government jobs be increased?
Arguments:
I. Yes, other countries have decided so long before.
II. Yes, it is the actual demand of lakhs of employees.

7. Statement:
Should new universities be established in India?
Arguments:
I. No, we have not yet achieved the target of literacy.
II. No, we have to face the problems of highly educated but unemployed youths.

8. Statement:
Should small States be formed out of bigger States in India?
Arguments:

Analytical Reasoning #3094, Sector 37D, Chandigarh. E-Mail: info@gjtutorial.com Page 12


Institute for All Competitive Exams
I. Yes, there will be administrative convenience.
II. No, it will be a danger to national integration.
9. Statement:
Should all the government owned educational institutions be given to private sector?
Arguments:
I. Yes, there will be up gradation of educational standard in these institutions.
II. Yes, the educational standard of these institutions will decrease.

10. Statement:
Should the organisations like the UNO be abolished?
Arguments:
I. The cold war is going to an end so there is no role of such organisations.
II. No, in the absence of these organisations, there will be a world war.

11. Statement:
Should there be complete ban on manufacture of fire-crackers in India?
Arguments:
I. No, this will render thousands of workers jobless.
II. Yes, the fire-cracker manufacturers use child labour to a large extent.

12. Statement:
Should the tuition fees in all post graduate courses be hiked considerably?
Arguments:
I. Yes, this will bring in some sense of seriousness among the students and will improve the
quality.
II. No, this will force the meritorious poor students to stay away from post-graduate courses.

13. Statement:
Should India allow the multinationals to explore the resources available under sea in its vast
economy zone?
Arguments:
I. Yes, India does not have enough technical and financial resources to conduct such explorations.
II. No, this will endanger the sovereign status of the country.

14. Statement:
Should the private companies be allowed to operate passenger train services in India?
Arguments:
I. Yes, this will improve the quality of service in Indian Railways as it will have to face severe
competition.
II. No, the private companies may not agree to operate in the non-profitable sectors.

15. Statement:
Should school education be made free in India?
Arguments:
I. Yes, this is the only way to improve the level of literacy.
II. No, it will add to the already heavy burden on the exchequer.

16. Statement:
Can psychiatrists treat maniac permanently?
Arguments:
I. Yes, they have all types of medicines.
II. No, the instincts keep on returning off and on.

Analytical Reasoning #3094, Sector 37D, Chandigarh. E-Mail: info@gjtutorial.com Page 13


Institute for All Competitive Exams
17. Statement:
Are marriages based on well-matched horoscopes more successful than other marriages?
Arguments:
I. Yes, there is no possibility of rifts or ups and downs as the horoscopes have already been studied
well.
II. No, such marriages are rather dull.

18. Statement:
Is death perfect cure to the physical sufferings?
Arguments:
I. Yes, when there is no body left, how can there be any ailment.
II. No, deep physical sufferings reach the spirit and must be continuing in same intensity after
death.

19. Statement:
Are the fabulous prices demanded by the art dealers for the original paintings of old masters
justified?
Arguments:
I. Yes, those are unattainable antique pieces of art, hence, worth their price for the collectors of art.
II. No, modern painters can paint as well if not better than them and for much less price.

20. Statement:
Should age of retirement be brought down?
Arguments:
I. Yes, such a decision on the part of the government would open new job opportunities to
youngsters.
II. No, people often marry late and their children don't get settled early.

21. Statement:
Should words like ―smoking is injurious to health‖ essentially appear on cigarette packs?
Arguments:
I. Yes, it is a sort of brain-wash to make the smokers realise they are inhaling poisonous stuff.
II. No, it hampers die enjoyment of smoking.

22. Statement:
Should hunting be banned?
Arguments:
I. Yes, it has been proved to be a definite environmental hazard.
II. No, what will hunters do?

23. Statement:
Does synonym mean exactly the same?
Arguments:
I. Yes, a rich language has many words for one thing.
II. No, each so called synonym is different from the other word and if used in its place, it changes
the meaning slightly.

24. Statement:
Should the age for marriage for boys be increased to 30 years?
Arguments:
I. No, it is arbitrary.
II. Yes, it may solve the population problem..

Analytical Reasoning #3094, Sector 37D, Chandigarh. E-Mail: info@gjtutorial.com Page 14


Institute for All Competitive Exams
25. Statement:
Will die newly elected members fulfil their promises?
Arguments:
I. Yes, otherwise their very existence will be in danger.
II. No, elected members never seem to remember their promises and commitments.

26. Statement:
Is murder better than committing suicide?
Arguments:
I. Yes, getting those who are responsible for problems out of way is better than sacrificing.
II. No, taking somebody's life or one's own life can never be right and there is always a third course
to be taken.

27. Statement:
Should MLA‘s and MP‘s be allowed to change parties?
Arguments:
I. Yes, every individual in democratic set up is free to leave or join any party any time.
II. No, they should remain faithful to the party which nominated them as its candidate.

28. Statement:
Married persons suffer less from psychological illness as compared to the single person.
Arguments:
I. Emotions in the case of married persons find adequate expression.
II. Married partners help each other.

29. Statement:
Do children adopted from poor families love their new rich parents like their own poor ones?
Arguments:
I. Yes, because they are loved, cared for and provided with comforts of life.
II. No, because in the criminal world, they become selfish and hard-boiled.

30. Statement:
Do scientists believe in religion?
Argument:
I. Yes, because science as a profession does not come in the way of faith.
II. No, because religion and faith do not go together.

Analytical Reasoning #3094, Sector 37D, Chandigarh. E-Mail: info@gjtutorial.com Page 15


Institute for All Competitive Exams

Descriptive Answers (Statement and Arguments)

1 (d):
None of the arguments is supported by a clear logic hence both the arguments are weak.
2 (e):
It is very known fact that unless you create awareness through advertisements about your product,
you lag behind from your competitors.
Contrary to this, heavy cost on advertisements adds to your product. Hence, both the arguments
are strong.
3 (e):
Both the arguments are strong. The high salary of executives in multinational companies would
definitely create unhealthy competition and consequently our companies, which are not rich, will
be in difficulties. Secondly, such ceiling will discourage multinationals to take interest in our
economy and encourage our companies.
4 (d):
Both the arguments are weak. The argument, that Govt's coffer can be filled only with the tax on
agriculture, is totally irrelevant. And secondly, it cannot be said that all the eighty percent rural
population are poor.
5 (a):
Considering India's economy, unscheduled holidays will hamper national progress. Hence,
argument I is strong.
6 (b):
Things successful in other countries may not necessarily succeed in our country hence argument I
is weak. Since, it is the demand of lakhs of employees, it is justified. Hence, argument II is strong.
7 (d):
Both the arguments are weak. There is no relation between the establishment of universities and
the achievement of the literacy target. Secondly, it appears illogical that due to establishment of
more universities, we shall have highly educated and unemployed youths. Hence, both the
arguments are weak.
8 (e):
Both the arguments are strong because there will be administrative convenience by forming small
states out of bigger states in India. Secondly, it may also lead to a danger to the national
integration.
9 (a):
Private sector is supposed to be more disciplined and efficient than Govt, sector, hence argument I
is strong.
10 (e):
Argument I is strong because the presence of UNO ensures friendly relation among various
countries. Argument II is also strong because now-a-days the presence of UNO is felt inevitable.
11 (e):
Fire-cracker industries employ thousands of workers; hence ban on such industries will render
thousands of workers jobless. Secondly, ban on industries is desirable because child labour
practice very much exists in such industries.
12 (b):
Argument I is irrelevant. Argument II is strong because poor and deserving students will be
affected by such hike in the fee.
13 (a):
Only argument I is strong because India has the abundant resources but does not have technical
skill for exploration. So India should go for foreign help. Secondly, India's sovereign status does
not get affected at all by doing so.
14 (a):

Analytical Reasoning #3094, Sector 37D, Chandigarh. E-Mail: info@gjtutorial.com Page 16


Institute for All Competitive Exams
Argument I is strong as entry of private sector will certainly improve the services of existing Govt,
sector because of the healthy competition. Argument II is weak because we are here concerned
with inviting the private company in Railways sector only.
15 (b):
Argument I is not strong because only making education free does not lead to absolute literacy.
Argument II is strong as it will put heavy burden on the exchequer.
16 (d):
None of the arguments is strong argument.
17 (d):
None of the arguments is supported by a valid logic; hence both the arguments are weak.
18 (d):
None of the arguments is supported by a valid logic; hence both the arguments lack the valid and
strong logic.
19 (a):
Argument I is strong. Fabulous prices for old master-piece are demanded because of their
originality and innovative hard efforts.
20 (e):
Both the arguments are strong because each of the arguments deals with the statement quite
logically.
21 (a):
Argument I is strong because the smokers will be cautioned of its ill-effects by such wordings.
Argument II is a weak argument as such wordings do, in no way, reduce the enjoyment of smoking.
22 (a):
Logically, only argument I is strong.
23 (b):
Second argument is strong because synonyms do not have exact meaning of given word. It is
nearest in the meaning to the word.
24 (d):
Both the arguments are weak.
25 (e):
Both the arguments are strong. Elected members have to face the electorates after each completion
of their term as their existence is decided by people. Secondly, at the time of elections, so many
commitments are made which are hardly fulfilled.
26 (b):
Argument II is strong because one should not become criminal by murdering other so as to wipe
out person responsible for problem.
27 (e): Both the arguments are strong. Each of the arguments is supported by a valid logic.
28 (e): Both, the arguments are strong as each of them is supported by strong logic.
29 (a):
If poor children are loved, cared for and provided with comforts of life, they will definitely respond
strongly and positively in term of respect to their parents. Argument II is not strong.
30 (a): Argument I is strong because profession and religion are altogether different. Argument II
is irrelevant

Analytical Reasoning #3094, Sector 37D, Chandigarh. E-Mail: info@gjtutorial.com Page 17


Institute for All Competitive Exams
Statement and Assumptions

These type of questions have a Statement followed by two or more assumptions. The candidate is
required to assess the given Statement and decide which of the given assumptions is implicit in the
Statement. Whenever we communicate with others, we make several Statements. Therefore,
Statement is a part of our speech or what we speak to others. Now, in communication, most of the
things remain unsaid. Hence, an assumption is something which is assumed supposed and taken
for granted.

Examples:
Here are some examples that will clarify the concept of assumption and statement.

1.
Statement:
Detergents should be used to clean clothes.
Assumptions:
I. Detergents form more leather.
II. Detergents help to dislodge grease and dirt.
Solution:
Detergents are being recommended to clean clothes. This implies that they must be having the
ability to clean. Now, ability to clean means the ability to dislodge grease and dirt, it does not mean
the ability to form more lather. Hence, Statement II is implicit, I is not implicit.

2.
Statement:
Love marriages mostly end in divorce.
Assumptions:
I. Love marriages do take place.
II. There are cases of divorce.
Solution
It is very clear from the Statement that divorce takes place when love marriage takes place. Hence,
both the Statements are valid.

3.
Statement:
To pass the examination, you have to practice very hard.
Assumptions:
I. Passing the examination is desirable.
II. Hard practice leads to success.
Solution
The advice is given on the behaviour that should be followed to pass the exam. This implies that
passing exams is necessary. So assumption I is implicit. Passing the exam is a type of success. And
according to the Statement, it may be achieved by hard work, hence assumption II is implicit.

4.
Statement:
A's advice to B - ―If you want to study Accounts, join institute Y‖.
Assumptions:
I. Institute Y provides good Accounts education.
II. B listens to A's advice.
Solution

Analytical Reasoning #3094, Sector 37D, Chandigarh. E-Mail: info@gjtutorial.com Page 18


Institute for All Competitive Exams
If A advises B to join a particular institute, A must have assumed that the particular institute was a
good institute. Secondly, while advising B, A must also have thought that B would listen to A's
advice. Hence, both the assumptions are valid.

5.
Statement:
Of all the TV sets manufactured in India, X brand has the largest sale.
Assumptions:
The sale of all the TV sets manufactured in India is known.
Solution
It is given in the Statement that of all TV sets in India, X brand has the largest sale; hence it must
have been assumed in the Statement that the sale figures of all brands are known.

6.
Statement:
―Use T - make iron in building construction‖ - an advertisement.
Assumptions:
T - make iron is the only iron used in building construction.
Solution
The use of word only makes the assumption invalid because no such expression is implicit in the
Statement. Hence, assumption is invalid.

7.
Statement:
Since certain sections of society are going to be unhappy whenever reforms are implemented, there
is little that can be done to prevent it except abandoning it.
Assumptions:
Some people would not like the implementation of reforms.
Solution
At first look, the assumption appears to be valid. But on a closer look, it is observed that
assumption is merely a reiteration or re Statement of the Statement. And such assumption will be
an invalid assumption.

8.
Statement:
Patient's condition would improve after this operation.
Assumptions:
I. The patient can be operated upon in this condition.
II. The patient can not be operated upon in this condition.
Solution
The Statement is based on die assumption that patient can be operated upon in this condition.
Hence, assumption I is implicit.

9.
Statement:
The best evidence of India's glorious past is the growing popularity of Ayurvedic medicines in the
west.
Assumptions:
I. Ayurvedic medicines are not popular in India.
II. Allopathic medicines are more popular in India.
Solution
Statement given in the question states only about the place which Ayurvedic medicine had
occupied in the past on account of its increasing popularity in west. And this signifies neither non -
Analytical Reasoning #3094, Sector 37D, Chandigarh. E-Mail: info@gjtutorial.com Page 19
Institute for All Competitive Exams
popularity of Ayurvedic medicines in India nor speaks of popularity of Allopathic medicines.
Hence, none of the assumptions is implicit in the Statement.

10.
Statement:
The anti - reservation movement was taken up by all the States.
Assumptions:
I. If one State starts a movement, it is customary for all the other states to follow it.
II. A major portion of the youths and employees all over the country were not in favour of
reservation.
Solution
The anti - reservation movement could get acceptability by all the States only because major
portion of the youths and employees were not in favour of the reservation. Statement is not based
on the assumption that it becomes customary for the other States to follow a movement started by
one State. Hence, only assumption II is implicit.

11.
Statement:
―If you want to give any advertisement, give it in newspaper X‖ - A tells B.
Assumptions:
I. B wants to publicise his products.
II. Newspaper X has a wide circulation.
Solution
The word 'if in the Statement denotes that B may or may not be willing to publicise his products.
Statement further suggests that advertisement should be given in newspaper X. It might have been
assumed that X will help advertise to large Section of readers. In other words, newspaper X has a
wide circulation. So, only assumption II is implicit.

Analytical Reasoning #3094, Sector 37D, Chandigarh. E-Mail: info@gjtutorial.com Page 20


Institute for All Competitive Exams

Practice Questions: Statement and Assumptions I

Directions: In each question below is given a Statement followed, by two assumptions numbered
I and II. An assumption is something supposed or taken for granted. You have to consider the
Statement and the following assumption and decide which of the assumption is implicit in the
Statement Give answer
(a) If only assumption I is implicit.
(b) If only assumption II is implicit.
(c) If either I or II is implicit.
(d) If neither I nor II is implicit.
(e) If both I and II are implicit.

1.
Statement:
It is desirable to put the child in die school at the age of 5 or so.
Assumptions:
I. At that age the child reaches appropriate level of development and ready to learn.
II. The schools do not admit children after six years of age.

2.
Statement:
Even with the increase in the number of sugar factories in India, we still continue to import sugar
Assumptions:
I. The consumption of sugar per capita has increased in India.
II. Many of the factories are not in a position to produce sugar to their fullest capacity.

3.
Statement:
―You must refer to learn to dictionary if you want to become a good writer‖ - A advises B.
Assumptions:
I. Only writers refer to the dictionary.
II. All writers, good or bad, refer to the dictionary.

4.
Statement:
If it does not rain throughout this month, most farmers would be in trouble this year.
Assumptions:
I. Timely rain is essential for farming.
II. Most of the farmers are generally dependent on rains.

5.
Statement:
A advises B, ―If you want to study English, join Institute Y‖.
Assumptions:
I. B listens to A's advice.
II. Institute Y provides good coaching for English

6.
Statement:
The leader of the main opposition party asserted that the call for chakka jam turned out to be a
great success in the entire State.

Analytical Reasoning #3094, Sector 37D, Chandigarh. E-Mail: info@gjtutorial.com Page 21


Institute for All Competitive Exams
Assumptions:
I. The people in future will support the main opposition party.
II. People probably are convinced about the reason behind the chakka jam strike call.

7.
Statement:
Govt, aided schools should have uniformity in charging various fees.
Assumptions:
I. The Government's subsidy comes from the money collected by way of taxes from people.
II. The Government, while giving subsidy, may have stipulated certain uniform conditions
regarding fees.

8.
Statement:
Unemployed allowance should be given to all unemployed Indian youths above 18 years of age.
Assumptions:
I. There are unemployed youths in India who need monetary support.
II. The Government has sufficient funds to provide allowance to all unemployed youth.

9.
Statement:
Everybody loves reading adventure stories.
Assumptions:
I. Adventure stories are the only reading material.
II. Nobody loves reading any other material.

10.
Statement:
Please consult before making any decision on exports from the company.
Assumptions:
I. You may take a wrong decision if you don't consult me.
II. It is important to take a right decision.

11.
Statement:
Postal rates have been increased to meet the deficit.
Assumptions:
I. The present rates are very low.
II. If the rates are not increased, the deficit can not be met.

12.
Statement:
If degrees are delinked with jobs, boys will think thrice before joining college.
Assumptions:
I. Students join college education for getting jobs.
II. A degree is of no use for getting a job.

13.
Statement:
The next meeting of the Board of the Institute will be held after one year.
Assumptions:
I. The Institute will remain in function after one year.
II. The Board will be dissolved after one year.
Analytical Reasoning #3094, Sector 37D, Chandigarh. E-Mail: info@gjtutorial.com Page 22
Institute for All Competitive Exams

14.
Statement:
A sentence in the letter to the candidates called for written exams - ―you have to bear your
expenses on travel etc.‖
Assumptions:
I. If not clarified, all the candidates may claim reimbursement of expenses.
II. Many organisations reimburse expenses on travel to candidates called for written examination.

15.
Statement:
The function will start at 3 p. m. You are requested to take your seats before 3 p. m.
Assumptions:
I. If the invitee is not in his seat before 3. p. m., the function will not start.
II. Function will start as scheduled.

16.
Statement:
A warning in a train compartment - ―To stop train, pull chain. Penalty for improper use Rs. 500.‖
Assumptions:
I. Some people misuse the chain.
II. On certain occasion, people may want to stop a running train.

17.
Statement:
Lack of stimulation in the four or five years of life can have adverse consequences.
Assumptions:
I. A great part of the development of observed intelligence occurs on the earliest years of life.
II. 50 percent of the measurable intelligence at 17 is predictable by the age of four.

18.
Statement:
The income tax rules need to be amended so that there is more incentive for the people to declare
their actual wealth.
Assumptions:
I. The income tax rules are not proper.
II. Some people do not declare their actual wealth.

19.
Statement:
―Considering that his ministry contains many hawala - tainted ministers, Mr. Rao has a moral
obligation to resign‖ - politician.
Assumptions:
I. The politician is not close to Mr. Rao.
II. The politician would like to inculcate moral principles in politicians.

20.
Statement:
―If we need to boost our exports, we can not ignore our ports.‖ - An economic analyst.
Assumptions:
I. Our ports are in a good shape.
II. Our ports are not in good shape.

Analytical Reasoning #3094, Sector 37D, Chandigarh. E-Mail: info@gjtutorial.com Page 23


Institute for All Competitive Exams

21.
Statement:
―There is concern about the health of the arrested VIPs but not about the harm they have done to
the nation's health‖ - A journalist.
Assumptions:
I. The VIPs are not more important than the nation.
II. The practice of being concerned over the health of the VIPs should be stopped.

22.
Statement:
In case of any difficulty about this case, you may contact our company's lawyer.
Assumptions:
I. Each company has a lawyer of his own.
II. The company's lawyer is thoroughly briefed about this case.

23.
Statement:
A good manager must draw the utmost from each worker.
Assumptions:
I. It is possible to get the maximum from each worker.
II. Managers are expected to get the best from their workers.

24.
Statement:
I can take you quickly from Kanpur to Lucknow by my car but then you must pay me double the
normal charges.
Assumptions:
I. Normally, it will take more time to reach Lucknow from Kanpur.
II. People want to reach quickly but they will not pay extra money for it.

25.
Statement:
A good executive has to be task - oriented as well as people - oriented.
Assumptions:
I. Some executives are people - oriented.
II. Some executives are not people - oriented.

26.
Statement:
Read this book to get detailed and most comprehensive information on this issue.
Assumptions:
I. The person, who wants this information, can read.
II. There are other books available on this issue.

27.
Statement:
Patient's condition would improve after operation.
Assumptions:
I. The patient can be operated upon in this condition.
II. The patient cannot be operated, upon in this condition.

28.
Analytical Reasoning #3094, Sector 37D, Chandigarh. E-Mail: info@gjtutorial.com Page 24
Institute for All Competitive Exams
Statement:
Let us announce attractive incentives for better performance.
Assumptions:
I. Incentive schemes do not work in long run.
II. The performance can be improved.

29.
Statement:
Why don't you invite Anthony for the Christmas party this year?
Assumptions:
I. Anthony is not from the same city.
II. Unless invited, Anthony will not attend the party.

30.
Statement:
If you offer peanuts, you get only monkeys.
Assumptions:
I. Money is not a good motivation.
II. Money cannot buy everything.

Analytical Reasoning #3094, Sector 37D, Chandigarh. E-Mail: info@gjtutorial.com Page 25


Institute for All Competitive Exams

Descriptive Answers (Statement and Assumptions I)

1. (a):
It is logical to say that at die age of 5, the child reaches appropriate level of development and ready
to learn. Hence, assumption I is very much implicit in the Statement. Assumption II is not implicit
as it is not relevant.
2. (d):
None of the assumptions is implicit in the Statement as there may be various reasons for the same.
3. (d):
None of the assumptions is implicit as nothing can be said definitely.
4. (e):
Since the Statement speaks of the essentiality and requirement of rain for farmers, hence both the
assumptions are implicit in die Statement.
5. (e):
Since A advises B to join institute Y to study English, hence it is assumed that institute Y provides
good coaching for English. Secondly, it is also assumed that B listens to A's advice.
6. (b):
Since the chakka jam call was accepted by and large in the entire state, it is very likely that people
are convinced about the reasons behind the chakka jam strike call. However, it is not necessary
that people will support the main opposition party in future without any strong and valid reasons.
7. (b):
Assumption I is not related with the Statement, hence is not implicit in the Statement. Assumption
II is implicit as it gives the solid base for the fact defined in the Statement.
8. (a):
Assumption. I, that Indian unemployed youths need monetary support, is the solid base for
providing allowance to all unemployed youths. However, assumption II, that Govt, has sufficient
funds, does not give valid reasoning. Hence, only assumption I is implicit.
9. (d):
None of the assumptions is implicit as each assumption lacks proper reasoning.
10. (e):
It is directed in the Statement that consultation is necessary before making any decision on the
export. It is, therefore, assumed that person directed may take a wrong decision. Secondly, it is
assumed in the light of the Statement that it is important to take a right decision.
11. (d):
On the basis of Statement, it is not certain that present rates are very low. We cannot say that
increasing the rates is the only way to meet the deficit. Hence, none of assumptions is implicit.
12. (a):
Job is the ultimate aim of every student. Hence, if degrees are delinked with jobs, students will
think twice before joining colleges. Hence, assumption I is implicit. Assumption II is contrary to it.
13. (a):
It is clear that the institute will remain in function after one year as it is given that Governing
Board meeting will be held after one year. However, we have no reason to say that Board will be
dissolved after one year. Hence, only assumption I is implicit.
14. (e):
Specific instruction has been given in writing to prevent chance of claim of reimbursement for
travelling expenses. Moreover, it is also assumed that instruction has been given because expenses
are paid by other organisation. Hence, both the assumptions are implicit in the Statement.
15. (b):
It is mentioned in the invitation that you are requested to take your seats before 3 p. m. It means
that function will start as scheduled. Hence, assumption n is implicit.

Analytical Reasoning #3094, Sector 37D, Chandigarh. E-Mail: info@gjtutorial.com Page 26


Institute for All Competitive Exams
16. (e):
It is clear that penalty will be imposed for improper use, hence it is clear that some people misuse
the chain. Hence, assumption. (I) and. (II), both are implicit.
17. (a):
What the Statement says has no direct correlation with assumption II. Assumption I directly
follows from the Statement. Percentage amount makes the assumption II doubtful as it can be
more or less than 50%.
18. (b):
It is given in the Statement that income tax rules need to be amended to tempt people to declare
their actual wealth. It does never mean that income tax rules are not proper. Proper in what
respect is not clear. However, assumption II describes the reasons for such amendment of income
tax rules. Hence, only assumption II is implicit.
19. (b):
In his opinion, politician basically intends to point towards moral principle of politics. Hence,
assumption II is implicit. Politician may or may not be close to Mr. Rao. So, assumption I is not
implicit.
20. (d):
None of the assumptions is implicit in the Statement.
21. (d):
None of the assumptions is implicit in the Statement.
22. (b):
It is not necessary that every company has a lawyer. So assumption I is not implicit. Since it is
advised in the Statement that for any difficulty about the case, consult company's lawyer. It is,
therefore, assumed that company's lawyer is thoroughly briefed about the case. Hence, assumption
II is implicit.
23. (e):
Both the assumptions are clearly implicit in the Statement.
24. (a):
From the word 'quickly‖ in the Statement, it is clearly assumed that normally it will take more time
to reach Lucknow from Kanpur. However, it cannot be said whether people will pay extra money
for reaching quickly or not. Hence, only assumption I is implicit in the Statement.
25. (d):
The Statement advises that an executive should be task - oriented as well as people - oriented. It is
based on the assumptions that some executives are only people - oriented and some executives are
only task - oriented. Hence, none of the assumptions follows.
26. (a):
It is assumed that the person instructed is capable of. reading the book. However, we can not say
anything about the other source of information. Hence, the information I is implicit.
27. (a):
Since doctor opines that patient's condition would improve after operation, it is, therefore,
assumed that patient can be operated upon in this condition.
28. (b):
Since as per Statement, incentive is meant for better performance, it is, therefore, assumed that
incentive will improve the performance.
29. (b):
We can not say that Anthony is not from the same city. But it is assumed that unless invited,
Anthony will not attend the party as invitation has been emphasised in the Statement. Hence, only
assumption II is implicit.
30. (d):
None of the assumptions is related with the Statement, and hence, not implicit in the Statement.

Analytical Reasoning #3094, Sector 37D, Chandigarh. E-Mail: info@gjtutorial.com Page 27


Institute for All Competitive Exams

Practice Questions: Statement and Assumptions II

Directions: In each question below is given a Statement followed by three assumptions


numbered I. II and III. An assumption is something supposed or taken for granted. You have to
consider the Statement and the following assumptions and decide which of the assumptions is
implicit in the Statement and then decide which of the answers (a), (b), (c), (d) and (e) is correct
answer and indicate it on the answer sheet.

1.
Statement:
―Z - TV, the only TV which gives the viewers chance to watch two programmes simultaneously‖ -
An advertisement.
Assumptions:
I. Sale of Z - TV may increase because of tire advertisement.
II. Some people may be influenced by the advertisement and buy Z - TV.
III. The sale of Z - TV may be on the downward trend.
(a) None is implicit
(b) Only II and III are implicit
(c) Only I and II are implicit
(d) All are implicit
(e) None of these

2.
Statement:
The company has recently announced series of incentives to the employees who are punctual and
sincere.
Assumptions:
I. Those who are not punctual at present may get motivated by the announcement.
II. The productivity of the company may increase.
III. The profit earned by the company may be more than the amount to be spent for the incentive
programmes.
(a) None is implicit
(b) All are implicit
(c) Only II and III are implicit
(d) Only I & II implicit
(e) None of these

3.
Statement:
The company has decided to increase the price of all its products to tackle the precarious financial
position.
Assumptions:
I. The company may be able to wipe out the entire losses incurred earlier by this decision.
II. The buyer may continue to buy its products even after the increase.
III. The company has adequate resources to continue production for few more months.
(a) None is implicit
(b) Only II and III are implicit
(c) Only I and III are implicit
(d) Only II is implicit
(e) None of these

Analytical Reasoning #3094, Sector 37D, Chandigarh. E-Mail: info@gjtutorial.com Page 28


Institute for All Competitive Exams
4.
Statement:
The State Govt, has unilaterally increased octroi by five percent all commodities entering into the
State without seeking approval of the Central Govt.
Assumptions:
I. The State Govt, may be able to implement its decision.
II. The Central Govt, may agree to support the State Govt's decision.
III. The State Govt, may be able to earn considerable amount through the additional octroi.
(a) All are implicit
(b) Only I and II are implicit
(c) None is implicit
(d) Only II and III are implicit
(e) None of these 5 statements

5.
Statement:
Ashok decided to leave office at 4. 00 p. m. to catch a flight to Bangalore departing at 6. 00 p. m.
Assumptions:
I The flight to Bangalore may be delayed.
II. He may be able to reach airport well before 6. 00 p. m.
III. He may get adequate time to reach for a vehicle to go to the airport.
(a) All are implicit
(b) Only II and III are implicit
(c) None is implicit
(d) Only II is implicit
(e) None of these

Directions: In each question below is given a Statement followed by three assumptions


numbered I, II and III. An assumption is something supposed or taken for granted. You have to
consider the Statement and the following assumptions and decide which of the assumptions is
implicit in the Statement, then decide which of the answers (a), (b), (c), (d) and (e) is correct
answer and indicate it on the answer sheet.

6.
Statement:
We must be prepared to face any eventuality and all the assignments must be completed as per
their schedule - Director tells the faculty members.
Assumptions:
I. There is possibility of any serious eventuality.
II. Dates are fixed for all the assignments.
III. Faculty members are supposed to complete the assignments.
(a) All are implicit
(b) Only II and III are implicit
(c) None is implicit
(d) Only III is implicit
(e) Only I is implicit

7.
Statement:
Training must be given to all the employees for increasing productivity and profitability.
Assumptions:
I. Training is an essential component of productivity.
II. Employee can't function effectively without proper training.
Analytical Reasoning #3094, Sector 37D, Chandigarh. E-Mail: info@gjtutorial.com Page 29
Institute for All Competitive Exams
III. Profitability and Productivity are supplementary to each other.
(a) None is implicit
(b) All are implicit
(c) Only III is implicit
(d) Either I or II is implicit
(e) Both I and III are implicit

8.
Statement:
In the recently held All India Commerce Conference, the session on 'Management of Service Sector
in India' surprisingly attracted large number of participants and also received very good media
coverage in the leading newspapers.
Assumptions:
I. People were not expecting such an encouraging response for service sector.
II. Service sector is not managed properly in India.
III. Media is always very positive towards service sector.
(a) Only I is implicit
(b) All are implicit
(c) None is implicit
(d) Either I or III is implicit
(e) Both II and III are implicit

9.
Statement:
The situation of this area still continues to be tense and out of control. People are requested to be
in their homes only.
Assumptions:
I. There had been some serious incidents.
II. People will not go to die office.
III. Normalcy will be restored shortly
(a) All are implicit
(b) None is implicit
(c) Only I is implicit
(d) Only I and III are implicit
(e) Only I and II are implicit

10.
Statement:
State Council for Teacher Education (SCTE) has laid down guidelines in respect of minimum
qualification for a person to be employed ' as a teacher in universities or in recognised institutions.
Assumptions:
I. The authorities will now appoint only qualified teachers.
II. Only qualified people will apply for the teaching post.
III. SCTE decides all the norms of educational qualification for teaching faculty.
(a) All are implicit
(b) None is implicit
(c) Only I is implicit
(d) Both I and II are implicit
(e) Only I and II are implicit

Directions: In each question below is given a Statement followed by three assumptions


numbered I. II and III. An assumption is something supposed or taken for granted. You have to
consider the Statement and the following assumptions and decide which of the assumptions is
Analytical Reasoning #3094, Sector 37D, Chandigarh. E-Mail: info@gjtutorial.com Page 30
Institute for All Competitive Exams
implicit in the Statement, then decide which of the answers (a), (b), (c), (d) and (e) is correct
answer and indicate it on the answer sheet.

11.
Statement:
Prakash decided to get the railway reservation in May for the journey he wants to make in July to
Madras.
Assumptions:
I. The railway issues reservations two months in advance.
II. There are more than one train to Madras.
III. There will be vacancy in the desired class.
(a) Only II and III are implicit
(b) Only I is implicit
(c) All are implicit
(d) Only I and II are implicit
(e) None of these

12.
Statement:
The school authority decided to open a summer school this year in the school compound for the
students in the age range of 7 - 14 years.
Assumptions:
I. All the students will attend the summer school.
II. All the parents will prefer to remain in city than going out of town for enabling their children to
attend the summer school.
III. Those, who cannot afford to go out of the station, will send their children to summer school.
(a) Only II is implicit
(b) Only II and III are implicit
(c) None is implicit
(d) All are implicit
(e) Only III is implicit

13.
Statement:
―X - Chocolate is ideal as a gift for someone you love‖. - An advertisement.
Assumptions:
I. People generally give gifts to loved one.
II. Such advertisements generally influence people.
III. Chocolate can be considered as a gift item.
(a) Only I and II are implicit
(b) Only I and III are implicit
(c) All are implicit
(d) Only II and III are implicit
(e) None of these

14.
Statement:
In view of the recent spurt in sugar prices in the open market, the Govt, has asked the dealers to
release vast quantity of imported sugar in the open market.
Assumptions:
I. The dealers will follow the government directive.
II. The sugar prices will come down.
III. The price of indigenous sugar will remain unchanged.
Analytical Reasoning #3094, Sector 37D, Chandigarh. E-Mail: info@gjtutorial.com Page 31
Institute for All Competitive Exams
(a) None is implicit
(b) Only I and II are implicit
(c) Only I and III are implicit
(d) All are implicit
(e) Only II and III are implicit

15.
Statement:
Prabodh wrote a second letter to his mother after a month when he did not receive any reply to the
first letter.
Assumptions:
I. Prabodh's mother did not receive the letter.
II The letter generally reaches within a fortnight.
III. His mother promptly sends reply to his letters.
(a) Only II and III are implicit
(b) Only I and II are implicit
(c) Only III is implicit
(d) Only I and III are implicit
(e) None of these

16.
Statement:
―I want to present a novel written by Prem Chand to Arnar on his birthday‖ - A tells B.
Assumptions:
I. Amar does not have any novel written by Prem Chand.
II. Novel is an acceptable gift for birthday.
III. A will be invited by Amar on his birthday.
(a) Only I and II are implicit
(b) Only I and III are implicit
(c) None of these
(d) Only II is implicit
(e) All are implicit

17.
Statement:
Unable to manage with the present salary, Tarun joined another company.
Assumptions:
I. The new company has better work environment.
II. The present company offers moderate pay packet.
III. The new company offers higher salary to all its employees.
(a) All are implicit
(b) None is implicit
(c) Only II is implicit
(d) Only II and III are implicit
(e) None of these

18.
Statement:
The Central Govt, has directed the State Governments to reduce govt. expenditure in view of the
serious resources crunch and it may not be able to sanction any additional grant to the States for
the next six months.
Assumptions:
I. The State Govts, are totally dependent on Central Govt, for their expenditure.
Analytical Reasoning #3094, Sector 37D, Chandigarh. E-Mail: info@gjtutorial.com Page 32
Institute for All Competitive Exams
II. The Central Govt, has reviewed the expenditure account of the State Govts.
III. The State Govts, will abide by the directions.
(a) Only II and III are implicit
(b) All are implicit
(c) None is implicit
(d) Only III is implicit
(e) None of these

19.
Statement:
―To make the company commercially viable, there is an urgent need to prune the staff strength and
borrow money from the financial institution.‖ - Opinion of a consultant.
Assumptions:
I. The financial institution lends money for such proposals.
II. The product of the company has a potential market.
III. The employees of the company are inefficient.
(a) Only II and III are implicit
(b) All are implicit
(c) None is implicit
(d) Only I and II are implicit
(e) Only I and III are implicit

20.
Statement:
The residents of the locality wrote a letter to the corporation requesting to restore normalcy in the
supply of drinking water immediately as the supply at present is just not adequate.
Assumptions:
I. The corporation may not take any action on the letter.
II. The corporation has enough water to meet the demand.
III. The water supply to the area was adequate in the past.
(a) Only II is implicit
(b) Only III is implicit
(c) Only I and II are implicit
(d) Only II and III are implicit
(e) None of these

21.
Statement:
―Buy Y T. V. for better sound quality.‖ - An advertisement.
Assumptions:
I. Y T. V. is die only T. V. in the market.
II. Y T. V. is the costliest.
III. People generally ignore such advertisements.
(a) Only I is implicit
(b) Only II is implicit
(c) Only III is implicit
(d) All are implicit
(e) None is implicit

22.
Statement:
―Look at her audacity. Madhu has not replied to my letter‖ - A tells B.
Assumptions:
Analytical Reasoning #3094, Sector 37D, Chandigarh. E-Mail: info@gjtutorial.com Page 33
Institute for All Competitive Exams
I. Madhu received his letter.
II. Madhu did not receive his letter.
III. The letter was sent by post
(a) Only I is implicit
(b) Only II is implicit
(c) Only III is implicit
(d) All are implicit
(e) None is implicit

23.
Statement:
―Put a notice on the board that all the employees should come in time to office.‖ - An officer tells
his assistant.
Assumptions:
I. All the employees come late.
II. Employees read such notice on the board.
III. Employees will follow the instructions.
(a) Only I and II are implicit
(b) Only III is implicit
(c) Only I and III are implicit
(d) Only II and III are implicit
(e) All are implicit

24. Statement
―If you want best training in computers, join our classes.‖ - An advertisement.
Assumptions:
I. People want training related with computers.
II. People read advertisement.
III. People want quality training.
(a) Only I is implicit
(b) All are implicit
(c) Only I and II are implicit
(d) Only II and III are implicit
(e) Only I and III are implicit

25.
Statement:
―In our report published last week, the name of the author was misspelt. We regret the error‖ - A
magazine editor.
Assumptions:
I. The name of the author was not easy to spell.
II. Publishing correct names of authors is not as important as the quality of the article.
III. Publishing correct names of authors is desirable.
(a) Only I is implicit
(b) Only II is implicit
(c) Only III is implicit
(d) I and III are implicit
(e) II and III are implicit

26.
Statement:
―As our business is expanding, we need to appoint more staff.‖ - Owner of a company informs his
staff.
Analytical Reasoning #3094, Sector 37D, Chandigarh. E-Mail: info@gjtutorial.com Page 34
Institute for All Competitive Exams
Assumptions:
I. The present staff is not competent.
II. More staff will further expand the business.
III. Suitable persons to be taken as staff will be available,
(a) Only I is implicit
(b) Only II is implicit
(c) Only III is implicit
(d) All are implicit
(e) None is implicit

27.
Statement:
―Wanted a two bedroom flat in the court area for immediate possession.‖ - An advertisement.
Assumptions:
I. Flats are available in court area.
II. Some people will respond to the advertisement.
III. It is a practice to give such an advertisement.
(a) None is implicit
(b) Only I and II are implicit
(c) Only II is implicit
(d) All are implicit
(e) None of these

28.
Statement:
A group of friends decided to go for a picnic to Avon during the next holiday season to avoid crowd
of people.
Assumptions:
I. Generally many people do not go to Avon.
II. People prefer other spots to Avon.
III. Many people do not know about Avon.
(a) All are implicit
(b) Only II is implicit
(c) Only I and II are implicit
(d) Only I and III are implicit
(e) None of these

29.
Statement:
The Telephone Company informed the subscribers through a notification that those, who do not
pay their bills by the due date, will be charged penalty for every defaulting day.
Assumptions:
I. Majority of the people may pay their bills by the due date to avoid penalty.
II. The money collected as penalty may set off the losses due to delayed payment.
III. People generally pay heed to such notices.
(a) None is implicit
(b) Only I and III are implicit
(c) Only II is implicit
(d) Only II and III are implicit
(e) None of these

Analytical Reasoning #3094, Sector 37D, Chandigarh. E-Mail: info@gjtutorial.com Page 35


Institute for All Competitive Exams
30.
Statement:
Considering the tickets sold during the last seven days, the circus authorities decided to continue
the show for another fortnight which includes two weekends.
Assumptions:
I. People may not turn up on week days.
II. The average number of people who will be visiting circus will be more or less same as that of the
last seven days.
III. There may not be enough response at other places.
(a) None is implicit
(b) Only II is implicit
(c) Both I and II are implicit
(d) All are implicit
(e) None of these

Analytical Reasoning #3094, Sector 37D, Chandigarh. E-Mail: info@gjtutorial.com Page 36


Institute for All Competitive Exams

Descriptive Answers (Statement and Assumptions II)

1. (c):
The advertisement as described in the Statement is meant to influence the buyers and thus
increasing sale of the television. Hence, assumption I and II are implicit.
2. (d):
Incentive is an award for the punctuality and sincerity and will work as a motivation force. This
will definitely increase the productivity of the company. However, profitability cannot be predicted
definitely as it depends on other factors.
3. (a):
None of the assumptions is implicit in the Statement. Assumption I is not implicit because the
word 'entire' makes the assumption doubtful. Assumption II is not implicit because neither we
know the quality of the product nor the market for this product. Assumption III is not implicit
because we cannot say anything about the quantity of resources of the company.
4. (a):
State Govt, decision to increase octroi by 5 percent is based on the assumption that State Govt, is
competent to implement the decision and also get the consent of Central Govt. The hike is in view
of earning additional amount.
5. (b):
Ashok has decided to keep a margin of 2 hours m advance assuming that he may be able to reach
the airport well before 6. 00 p. m. and also get adequate time to search for a vehicle to go to the
airport.
6. (a):
All the assumptions are clearly implicit in the Statement.
7. (d):
Training has been necessitated in the Statement, i.e., it is related to the efficiency and effectiveness
of an employee. Therefore, assumption I and II are implicit in the Statement.
8. (a):
The use of word 'surprisingly' in the Statement clearly implies that people were not expecting such
a good response. Hence, assumption I is implicit. Assumption II and III are not related with the
Statement, and hence do not follow in the Statement.
9. (d):
The situation is tense, it means that some serious incidents had been occurred. Secondly, it is given
that situation still continues to be tense, it means that people may not go to the office. However, we
cannot say when the normalcy will be restored. Hence, only assumption I and II follow.
10. (a):
All the assumptions are clearly implicit in the Statement.
11. (b):
In the Statement, nothing has been given about the numbers of trains and vacancy for reservation.
Therefore, assumption II and III are not implicit. - However, - it is given that Prakash takes
reservation for July in May clearly indicates that there is provision of two months in advance. So,
assumption I is implicit.
12. (c):
The use of the word 'all' in assumptions I and II makes the assumption doubtful. And also we
cannot say that those, who cannot afford to go out of the station, will send their children to
summer school. Hence, none of the assumption is implicit.
13. (c):
The Statement implies that gifts are given for loved ones. It is also assumed that such
advertisement generally influence, people. Since chocolate has been recommended in the
advertisement, hence it is an ideal gift. Therefore, all the assumptions are implicit.
14. (b):

Analytical Reasoning #3094, Sector 37D, Chandigarh. E-Mail: info@gjtutorial.com Page 37


Institute for All Competitive Exams
Govt, has' issued orders means that the dealers will follow it. Secondly, Govt, directive to release
sugar is meant to bring down the prices of sugar. However, the impact on the indigenous sugar
price cannot be determined. Hence, only assumptions I and II are implicit.
15. (c):
Probodh wrote the second letter to his mother only when he did not receive the reply from his
mother. This fact implies that his mother promptly sends reply to his letters. Hence, only
assumption III is implicit.
16. (e):
A wants to present a book on B's birthday. It means that book is an acceptable gift. And also
assumption III is implicit. But we do not have this combination as our alternative. Hence, option
(e) is our correct answer.
17. (c):
Tarun has left the company in view of the meagre salary. Hence, assumption II is implicit.
However, we cannot say anything about the work environment and salary of all the employees.
18. (a):
The use of the word 'totally‖ makes the assumption doubtful. It is given that Central Govt, has
directed State Govt, to reduce its expenditure account and also State Govt, will abide by the
directives.
19. (d):
Consultant in his opinion emphasises the need for mobilising the staff and raising the funds. This
means that the product of the company has a potential market and also that the financial
institutions provide money for such proposals.
20. (b):
Complaint of the residents relating to the present water supply implies that the water supply was
adequate in the past. Hence, assumption III is implicit.
21. (e):
Clearly, none of the assumptions is implicit in the Statement.
22. (a):
A has categorically stated that Madhu has not replied to his letter, it means that she has received
his letter.
23. (d):
Assumption II is implicit because instruction is to display the notice on the board. Secondly, it is
an office order. Hence, assumption III is also implicit.
24. (b):
It is assumed in the Statement that people want quality training in computers. Secondly, it is an
advertisement. Hence, people read advertisement. Hence, all assumptions are implicit.
25. (c):
It is only said in the Statement that the name was misspelt and that is regretted. It would not be
proper to draw conclusions such as I or II. Only III is implicit. If it were not desirable, the name
would be spelt correctly.
26. (e):
Clearly, none of the assumptions is implicit.
27. (c):
Advertisement has been given assuming that some people will respond to the advertisement.
Hence, only assumption II is implicit. However, assumption I and III are doubtful.
28. (c):
It is given in the Statement that friends have decided to go for picnic to avoid crowd. This implies
that many people do not go to Avon. Hence, I and II are implicit. Now we cannot say that reason
for the Avon being non - crowdy is people's unacquaintance with this spot. It may be due to other
reasons also.
29. (b):

Analytical Reasoning #3094, Sector 37D, Chandigarh. E-Mail: info@gjtutorial.com Page 38


Institute for All Competitive Exams
It cannot be definitely said that for what purpose the money collected as penalty will be used.
However, assumptions I and III are in the line of the Statement and hence, implicit in the
Statement.
30. (b):
The response of the public to the circus shows during the last seven days is reasonably good. It is,
therefore, assumed by the authority to gather the similar responses during the extended days.
Hence, assumption II is implicit.

Analytical Reasoning #3094, Sector 37D, Chandigarh. E-Mail: info@gjtutorial.com Page 39


Institute for All Competitive Exams
Practice Questions: Statement and Assumptions III

Directions: In each question below is given a statement followed by two assumptions numbered I
and II. An assumption is something supposed or taken for granted. You have to consider the
statement and the following assumptions and decide which of the assumptions is implicit in the
statement. Give answer:
1. If only assumption I is implicit;
2. If only assumption II is implicit;
3. If either assumption I or II is implicit;
4. If neither assumption I nor II is implicit; and
5. If both assumption I and II is implicit

1.
Statements:
To mitigate the impact of asymmetric migration of trained people from the
developing countries, the UNESCO may catalyse a more symmetric and closer interaction of
science and technology personnel across the world.
Assumptions:
I. Migration of trained people from developing countries is a problem for developing countries.
II. UNESCO is an international organisation which solves the problems of needy countries.

2.
Statements:
The injured person was brought to the hospital, where he was declared
dead by the doctors.
Assumptions:
I. An injured person is always taken to a hospital, where he is declared dead by the doctors.
II. Only doctors can understand whether a person is dead or alive, medically.

3.
Statements:
The BIFR has declared Company X as sick.
Assumptions:
I. The BIFR has the right to declare a company sick.
II. The company must actually be sick.

4.
Statements:
The chandelier fell down and broke into pieces‘, said the dejected curator of the museum.
Assumptions:
I. The curator was sad at the incident having occurred.
II. The chandelier must have been very costly.

5.
Statements:
Norway is called the land of the mid-night Sun.
Assumptions:
I. Norwegians can see the Sun at mid-night in their country.
II. The sun never sets in Norway.

Directions: In each question below is given a statement followed by two assumptions numbered I
and II. An assumption is something supposed or taken for granted. You have to consider the
Analytical Reasoning #3094, Sector 37D, Chandigarh. E-Mail: info@gjtutorial.com Page 40
Institute for All Competitive Exams
statement and the following assumptions and decide which of the assumptions is implicit in the
statement. Give answer
1. If only assumption I is implicit.
2. If only assumption II is implicit.
3. If either assumption I or II is implicit.
4. If neither assumption I nor II is implicit.
5. If both assumption I and II is implicit.

6.
Statements:
The accident occurred because the car collided with a truck travelling at a very slow speed.
Assumptions:
I. The car must have been travelling at a speed greater than that of truck.
II. The car must have hit the truck from the back.

7.
Statements:
The injured person was brought to the hospital, where he was declared dead by the doctors.
Assumptions:
I. An injured person is always taken to a hospital.
II. Only doctors can understand whether a person is dead or alive, medically.

8.
Statements:
Animals are our mute friends.
Assumptions:
I. Only mute animals are our friends.
II. All animals are our friends.

9.
Statements:
The fighter plane had to crash-land an hour after it took off.
Assumptions:
I. The plane might have been hit by a bird.
II. The plane might have run short of fuel.

10.
Statements:
The President, in his televised address to the nation, asked his fellow-countrymen to maintain
peace and harmony.
Assumptions:
I. A communal riot had broken in the country.
II. The country has a presidential system of governance.

Directions: In each question below is given a statement followed by two assumptions numbered I
and II. An assumption is something supposed or taken for granted. You have to consider the
statement and the following assumptions and decide which of the assumptions is implicit in the
statement. Give answer
1. If only assumption I is implicit.
2. If only assumption II is implicit.
3. If either assumption I or II is implicit.
4. If neither assumption I nor II is implicit.
5. If both assumption I and II is implicit.
Analytical Reasoning #3094, Sector 37D, Chandigarh. E-Mail: info@gjtutorial.com Page 41
Institute for All Competitive Exams
11.
Statements:
Most of the defence personnel are capable of choosing between right and wrong since they have
basic qualifications.
Assumptions:
I. Basic qualification is necessary to differentiate between right and wrong.
II. Some defence personnel are unable to differentiate between right and wrong.

12.
Statements:
As growth in bank deposits slows down and government and corporate step up borrowing, interest
rates could go up later this year.
Assumptions:
I. The depositing of money in bank depends a lot on the interest rate given on the sums deposited.
II. Interest rates went up last year.

13.
Statements:
Unless the officials do their duty honestly and with the spirit to serve the nation, the constitution
of any number of safety boards amounts to nothing but paper work.
Assumptions:
I. Safety boards are useless.
II. Employees of the department can prevent accidents.

14.
Statements:
A film is no different from a poem, a story or a painting. Can one think of erasing a painting for
instance? Or, ask a poet to delete some lines of his work? Sounds ridiculous! It is the same thing
about a film: one must see it in the form the director wants us to see it.
Assumptions:
I. Boards usually pressurise directors of a film to cut some shots.
II. Filmmakers should have the right to show whatever they want.

15.
Statements:
Mahatma Gandhi would have been pained to see that leaders in his country have been reduced to
high living and simple thinking, instead of the other way round.
Assumptions:
I. Mahatma Gandhi is father of the nation.
II. Mahatma Gandhi believed in simple living and high thinking.

Directions: In each question below is given a statement followed by two assumptions numbered I
and II. An assumption is something supposed or taken for granted. You have to consider the
statement and the following assumptions and decide which of the assumptions is implicit in the
statement. Give answer
1. If only assumption I is implicit.
2. If only assumption II is implicit.
3. If either assumption I or II is implicit.
4. If neither assumption I nor II is implicit.
5. If both assumption I and II is implicit.

Analytical Reasoning #3094, Sector 37D, Chandigarh. E-Mail: info@gjtutorial.com Page 42


Institute for All Competitive Exams
16.
Statements:
Teachers should strive to acquire learning competence, develop commitment to objectives and
improve performance.
Assumptions:
I. Now a days, teachers are money minded and have strayed away from their main objective.
II. The future of a country depends a lot on teachers.

17.
Statements:
Mr. Rupesh has already played the best part of his cricketing life and now he is just passing time.
Assumptions:
I. Mr Rupesh should now quit cricket.
II. Mr Rupesh does not want to quit cricket.

18.
Statements:
Customers need to protect themselves against adulteration with the help of various food
adulteration testing kits.
Assumptions:
I. Government seems unable to carry out the responsibility.
II. Certain cases of food adulteration have come to notice.

19.
Statements:
In India, whenever a disaster occurs it is time for the government to constitute a committee.
Assumptions:
I. Making committees is an unnecessary exercise and is meant to fool the public.
II. No committee‘s recommendations has been implemented in the past.

20.
Statements:
Except emergency services like ambulance, fire brigade and the police, no vehicle should be given
priority on the roads.
Assumptions:
I. There are some vehicles other than emergency services which are given priority on the roads.
II. People will not mind the disturbance in traffic caused by emergency services vehicles.

Answer key (Statement and Assumptions III)


1: 24141 6:12254 11: 21252 16:15541

Analytical Reasoning #3094, Sector 37D, Chandigarh. E-Mail: info@gjtutorial.com Page 43


Institute for All Competitive Exams
Statement and Conclusions

Statement and Conclusion is a mere extension of the Statement and Assumptions. You are
required to approach the questions for solution from a different angle. This approach basically
emphasises the need for being more systematic and logical in drawing inference.
In this type of questions statement is followed by conclusions instead of assumptions. They are
designed to test the candidates' ability to interpret a statement. Therefore, consequent effect of a
statement has to be analysed before reaching the correct conclusion.
Students are therefore required to go through the examples given here and read, the explanations
following each question. It will enable the students to understand the logical approach required to
draw valid conclusion.

Directions: In each of the following questions, a statement is given followed by two conclusions I
and II. Give answer
(a) If only conclusion I follows
(b) If only conclusion II follows
(c) If either I or II follows
(d) If neither I nor 11 follows
(e) If both I and II follow.

1. Statement:
Parents are prepared to pay price for an elite education to their children.
Conclusions:
I. All parents these days are very well off.
II. Parents have an obsessive passion for a perfect development of their children through good
schooling.

Solution:
It may be concluded from the statement that since parents want a perfect development of their
children through good schooling, therefore, they are prepared to pay any price for a good
education. But statement does not give sense of the parent being very well off. Hence, only
conclusion II follows. Therefore, our answer is (b).

2. Statement:
Interview panel may select a student who is neither possessing the abilities of desired level nor any
value and assumptions.
Conclusions:
I. Inclusion of experts in interview panel does not ensure that the selection will be made properly.
II. Interview procedure of admission has some limitations.

Solution:
The statement clearly means that inclusion of experts does not ensure proper selection. It also
indicates limitation of interview procedure for admission. Hence, both conclusions follow.
Therefore, our answer is (e).

3. Statement:
The constitution amendment carried out just last month prohibits the employment of child labour
in any organisation.
Conclusions:
I. The employees must now abide by this amendment to the constitution.
II. Children below 14 years will now be engaged in acquiring education.

Analytical Reasoning #3094, Sector 37D, Chandigarh. E-Mail: info@gjtutorial.com Page 44


Institute for All Competitive Exams
Solution:
It is clearly deduced from the statement that consequent upon the amendment prohibiting the
child labour, the employer must abide by the amendment. But it is not mentioned that amendment
gives any provision for acquiring education. Hence, only conclusion I follows. Therefore, our
answer is (a).

4. Statement:
In India, emphasis should be given more to agriculture engineering and technological fields rather
than on basic and pure science.
Conclusions:
I. India has achieved sufficient development in the field of basic and pure science.
II. In the past, the production sector of economy was neglected.

Solution:
The extent of the development in the field of basic and pure science cannot be ascertained from the
statement. Similarly, the production sector of economy was neglected in the past or not, cannot be
concluded. Hence, none of the conclusions follow the statement Therefore, our answer is (d).

5. Statement:
Any young man, who makes dowry as a condition for marriage, discredits himself and dishonours
womanhood.
Conclusions
I. Those, who take dowry in marriage, should be condemned by society.
II. Those, who do not take dowry in marriage, respect womanhood.

Solution:
Clearly the statement declares dowry as evil practice and reflects its demerits. Thus, conclusion I
follows. Also, it is given that those who take dowry, dishonour womanhood. This implies that
those, who do not take dowry, respect womanhood. So, conclusion II follows.
Hence, answer is (e).

Analytical Reasoning #3094, Sector 37D, Chandigarh. E-Mail: info@gjtutorial.com Page 45


Institute for All Competitive Exams

Practice Questions: Statement and Conclusions I

Directions: In each of the question given below, one or more statement is/are followed by
inferential conclusions. The conclusion, which can be derived without supposing anything else, i.e.
without adding anything extra to the statement (s) is your answer.

1. Statement:
The Education Secretary said that it was extremely necessary that the professional researchers
along with practical school teachers conduct the practical research.
Conclusions:
(a) Practicing teachers know the technique of research.
(b) Professional researchers are quite aware of the problems related to the school education.
(c) Educational Secretary was speaking to the school students.
(d) There is a communication gap between researchers and the practicing school teachers.
(e) School education is quite a specialised field and so long as teacher is not trained, the standard
of education cannot be qualitatively better.

2. Statement:
Vehicular source contributes 60% to the total pollution level of Delhi, as informed in the Rajya
Sabha today.
Conclusions:
(a) The court has ordered that the main source of pollution should be identified.
(b) The problem of pollution is high in the work list.
(c) In the other cities also, vehicular source contributes around the same percentage level of
pollution.
(d) This matter was raised first in the Lok Sabha.
(e) Not many towns in India face the problem of pollution.

3. Statement:
PVC shoe producers union has welcomed the announcement of relief on production surcharge
from Rs. 50 per pair to Rs 75 per pair.
Conclusions:
(a) All PVC shoe producers are members of this union.
(b) Relief on production surcharge of Rs 50 on every pair to shoes had been given only recently.
(c) Union had demanded an increase in relief.
(d) Govt will collect more production surcharge.
(e) Other producers will also demand relief on production surcharge.

4. Statement:
There are chances of hike in the electricity tariff soon. This proposal will bring about a sharp
increase for the industrial consumers and marginal increase for the domestic consumers.
Conclusions:
(a) At present there is a disparity in the electricity tariff between the domestic and the industrial
consumers.
(b) Industrial consumers consume more electricity than the domestic users.
(c) Consumption of electricity by the industrial consumer is comparatively lower than the domestic
user.
(d) After this hike, the electricity board will start making profit.
(e) If this proposal is accepted, then industrial output will get positively affected.

Analytical Reasoning #3094, Sector 37D, Chandigarh. E-Mail: info@gjtutorial.com Page 46


Institute for All Competitive Exams
5. Statement:
State government imposes a monthly tax on the salary of all the employees. The tax amount varies
according to the income slab. The Central Govt also imposes tax on the same income, which is
known as the income tax. This is against the theory of taxation.
Conclusions:
(a) A regular collection, irrespective of the income slab, by the Central govt, is the income tax.
(b)The Central Govt should not collect any tax on income.
(C) Income tax should not vary according to the income slabs.
(d)Tax should be imposed only once on an individual.
(e) The State Govt should not impose any tax on income.

6. Statement:
A forest has as many Sandal trees as it has Ashoka trees. Three-fourths of the trees are old and half
of the trees are at the flowering stage.
Conclusions:
(a) All Ashoka trees are at the flowering stage.
(b) All Sandal trees are at the flowering stage.
(c) At least one-half of the Ashoka trees are old.
(d) One half of the Sandal trees are at the flowering stage.
(e) None of these.

Directions: In each question below are given certain statements followed by some conclusions.
Choose the conclusion which follows from the given statement.

7. Statement:
1. Only students can participate in the race.
2. Some participants in the race are females.
3. All female participants in the race are invited for coaching.
Conclusions:
(a) All participants in the race are invited for coaching.
(b) All participants in the race are males.
(c) All students are invited for coaching.
(d) All the participants in the race are students.

8. Statement:
1. Shyam is not the father of Hari.
2. Hari is the son of Suresh.
3. Suresh has three sons.
Conclusions:
(a) Shyam is son of Suresh.
(b) Hari is the brother of Shyam.
(c) Suresh is the father of Hari.
(d) Shyam has no children.

9. Statement:
1. None but the rich can afford air travel.
2. Some of those who travel by air become sick.
3. Some of those who become sick require treatment.
Conclusions:
(a) All the rich persons travel by air.
(b) Those who travel by air become sick.
(c) All the rich persons become sick.
(d) All those who travel by air are rich.
Analytical Reasoning #3094, Sector 37D, Chandigarh. E-Mail: info@gjtutorial.com Page 47
Institute for All Competitive Exams
Directions: In each question below a statement is given followed by two conclusions numbered I
and II. You have to take the statement to be true. Read both the conclusions and decide which of
the two or both follow from the given statement. Give answer
(a) If only conclusion I follows.
(b) If only conclusion II follows.
(c) If either I or II follows.
(d) If neither I nor II follows.
(e) If both conclusions I and II follow.

10. Statement:
The nation X faced the increased international opposition due to its decision of performing eight
nuclear explosions.
Conclusions:
I. The citizens of the nation have favoured the decision.
II. Some powerful nations don't want that others may become powerful.

11. Statement:
The chief minister emphasised the point that the Govt, will try its best for the development of the
farmers and rural poor.
Conclusions:
I. The former Govt had not tried seriously for the development of these people.
II. This Govt will not try seriously for the development of urban poor.

12. Statement:
In India, emphasis should be given more on agriculture, engineering and technological fields
rather than on basic and pure science.
Conclusions:
I. India has achieved sufficient development in the field of basic and pure science.
II. In the past, the production sector of economy was neglected.

13. Statement:
The majority of Indian labourers belong to unorganised sector and most of them earn very low.
Conclusions:
I. The labourers belonging to organised sector have better benefits and stability. II. Some labourers
belonging to unorganised sector have regular and certain income.

14. Statement:
The multinational fast food chains have started their operation in India after facing many
problems but Association of Farmers is ready for competition.
Conclusions:
I. Association of Farmers are not supporting modernisation.
II. Association of Farmers are not ready -for competition with the multinational companies.

15. Statement:
Company can understand the problems of its customers by direct marketing where it gets a chance
to correct those problems. Thus, the customer becomes a reliable buyer.
Conclusions:
I. Responsible companies profit by direct marketing.
II. Direct marketing is the only key to success in business.

16. Statement:
Company 'X' has a record in making cameras with quality and modernisation to ensure that you
are losing not even a single shot despite bad weather.
Analytical Reasoning #3094, Sector 37D, Chandigarh. E-Mail: info@gjtutorial.com Page 48
Institute for All Competitive Exams
Conclusions:
I. No other company has any recognition in camera industry.
II. Any common man can take good photographs using camera 'X'.

17. Statement:
Though in terms of number of schools, the educational system has very much expanded but most
of them are not sufficiently equipped and are not able to touch the high water mark in education.
Conclusions:
I. We should provide good teachers and equipments to existing school now on ward.
II. Now it is not necessary to open new school.

18. Statement:
The top management has asked the four managers either to resign by tomorrow or face the order
of service termination. Three of them have resigned till this very evening.
Conclusions:
I. The managers who did not resign yesterday will resign tomorrow.
II. The management will terminate the service of one manager.

19. Statement:
Our security investments carry market risk. Consult your investment advisor or agent before
investing.
Conclusions:
I. One should not invest in securities.
II. The investment advisor calculates the market risk with certainty.

20. Statement:
In diabetes, there is an excess of sugar in the body. Our body needs sugar for energy. The quantity
of sugar in excess of body's requirements is excreted through urine.
Conclusions:
I. The excessive consumption of sugar is likely to lead to diabetes.
II. Consumption of sugar should be avoided.

21. Statement:
Today, out of the word population of several thousand million, the majority of men have to live
under Governments which refuse them personal liberty and the right to dissent.
Conclusions:
I. People are indifferent to personal liberty and right to dissent.
II. People desire personal liberty and right to dissent.

22. Statement:
He emphasised the need to replace the present training programme by other methods which will
bring out the real merit of the managers.
Conclusions:
I. It is important to bring out the real merit of the managers.
II. The present training programme does not bring out the real merit of the managers.

23. Statement:
The greatest need in India today is not for sophisticated gadgets but for programmes which will
provide employment to large number of people.
Conclusions:
I. There is an adequate number of sophisticated gadgets in India.
II. Emphasis is being laid on procuring sophisticated gadgets.

Analytical Reasoning #3094, Sector 37D, Chandigarh. E-Mail: info@gjtutorial.com Page 49


Institute for All Competitive Exams
24. Statement:
Company 'X' has marked the product. Go ahead and purchase it, if price and quality are your
considerations.
Conclusions:
I. The product must be good in quality.
II. The price of the product must be reasonable.

25. Statement:
In case of the outstanding candidates, the condition of previous experience of social work may be
waived by the admission committee for M.A. (Social work).
Conclusions:
I. Some of the students for M.A. (Social work) will have previous experience of social work.
II. Some of the students for M.A. (Social work) will not have previous experience of social work.

26. Statement:
It is almost impossible to survive and prosper in this world without sacrificing ethics and morality.
Conclusions:
I. Word appreciates some concepts but may not uphold it.
II. Concept of ethics and morality are not practicable in life.

27. Statement:
Any student, who does not behave properly while in the school, brings bad name to himself and
also to the school.
Conclusions:
I. Such students should be removed from the school.
II. Strict discipline does not improve behaviour of the students.

28. Statement:
To cultivate interest in reading, the school has made it compulsory from June 96 for each student
to read 2 books per week and submit a weekly report on the books.
Conclusions:
I. Interest in reading can be created by force.
II. Some students eventually will develop interest in reading.

29. Statement:
Modern man influences his destiny by the choice, he makes unlike in the past.
Conclusions:
I. Earlier there were fewer options available to them.
II. There was no desire in the past to influence the destiny:

30. Statement:
While presenting a stage show recently, the famous actor declared that he has a practice of either
taking full payment or none for his stage show.
Conclusions:
I. The actor has taken full payment for his recent stage show.
II. The actor did not take any money for his recent stage show.

Analytical Reasoning #3094, Sector 37D, Chandigarh. E-Mail: info@gjtutorial.com Page 50


Institute for All Competitive Exams

Descriptive Answers (Statement and Conclusions I)

1 (e):
Clearly, statement gives the sense that if practicing' school teachers accompany professional
researchers, and then teachers will be benefitted by the expertise of professionals. As a result, the
standard of education will be alleviated.
2 (b):
Problem of pollution was discussed in the Rajya Sabha because it was getting priority in the work
list.
3 (c):
Only conclusion (c) follows because relief on production surcharge follows a demand for the same
by union.
4 (b):
As per the statements, there is different treatment for industrial consumers and domestic
consumers. It is because industrial consumers consume more electricity than the domestic users.
5 (d):
Only conclusion (d) follows as tax should be imposed only once on an individual.
6 (c):
Supposing numbers of Sandal trees and Ashoka trees are 50 each. Therefore, total will be 100. Out
of which 75 trees are old. Hence, it is confirmed that at least one-half of the Ashoka trees are old.
7 (d):
It is given in the statement (1) that only student can participate in the race. Hence, it directly
implies that all participants in the race will be students.
8 (c):
From statement (2), it is very clear that Suresh is the father of Hari.
9 (d):
From statement (1), it is clear that all those who travel by air are rich.
10 (b):
Nuclear explosion is not the result of the favour by the citizens. Hence, conclusion (I) does not
follow. However, increase in the international opposition is due to the fact that some powerful
nations do not want that others may become powerful.
11 (d):
From the commitment of the present Govt., it cannot be concluded that former Govt, did not do
anything serious about the development nor does it mean that present Govt, will not do anything
for development of other sectors, hence neither of the conclusions follows.
12 (d):
Neither of the conclusions definitely follows from the statement because it cannot be concluded
from the statement that India has achieved sufficient development in the field of basis and pure
science. Nor does the statement mean that production sector in the past remained neglected.
13 (b):
It is given in the statement that most of the labourers belonging to unorganised sector earn very
low and uncertain, it means that there are some labourers who earn regular and certain income.
Hence conclusion (b) definitely follows from the statement.
14 (d):
Conclusion I is totally unrelated to the statement and conclusion II is contrary to the statement.
Hence, none of the conclusions follows.
15 (a):
The word 'only‖ in conclusion II makes it doubtful. Conclusion I is direct result of the statement, as
statement speaks of the merits of direct marketing.
16 (d):

Analytical Reasoning #3094, Sector 37D, Chandigarh. E-Mail: info@gjtutorial.com Page 51


Institute for All Competitive Exams
Conclusion I does not follow because companies other than 1C may also have recognition in
camera industry. Conclusion II does not follow because statement speaks of the quality of camera,
not of the cameraman. Hence, none of the conclusions follows.
17 (a):
Conclusion I follows because opening number of schools will not serve the very purpose of
education unless these schools are provided with good teachers and equipments. Conclusion II
does not follow as it is not the right solution of the problem.
18 (c):
In view of the top management order, either the remaining manager will resign or the
management will terminate the service of one manager. Hence, either I or II follows.
19 (b):
The investment consultants are acquainted with the market risks. Therefore, it is advisable to
consult them before investing your money in securities.
20 (a):
Avoiding consumption of sugar is not the right approach to avoid diabetes. Hence, conclusion II
does not follow. However, in the light of the statement, conclusion I follows as excessive sugar is
the most likely cause of diabetes.
21 (b):
It is clear from the statement that people desire personal liberty and right to dissent.
22 (e):
Both the conclusions follow from the statement. Conclusion I is the direct result of the statement.
Conclusion II follows because as per statement, there is need to replace present system with the
new one.
23 (d):
Statement says that greatest need in India is not for sophisticated gadgets but for programmes, but
this does not mean that there is an adequate number of sophisticated gadgets. Secondly, there is a
need for programmes but it cannot be concluded that emphasis is being laid on procuring
sophisticated gadgets. Hence, none of the conclusions follows.
24 (e):
It is clear from the statement that product marked by company 'X' has good quality and is available
at reasonable price. Hence, both conclusions follow.
25 (e):
Both the conclusions follow. The condition of waiver is incorporated because some of the students
for M.A. will have previous experience and some of the students will not have previous experience
of social work.
26 (e):
Conclusions I and II convey almost the same thing that principles related to ethics and morality
seem to be good but are not practicable in life. Hence, both the conclusions follow.
27 (d):
Conclusion I does not follow because removal of undisciplined student does not provide a
reasonable solution to the problem.
28 (b):
Interest of reading cannot be inculcated by force. Hence, conclusion I does not follow. However, a
gradual reading will develop a spontaneous inclination towards reading. Hence, conclusion II
follows.
29 (a):
There was desire in the past to influence the destiny but there were less options available that time,
hence conclusion (I) follows.
30 (c):
As per the condition given in the statement, the actor either will take the full payment for his
recent show or will not take any money for the same. Hence, either of the conclusions follows

Analytical Reasoning #3094, Sector 37D, Chandigarh. E-Mail: info@gjtutorial.com Page 52


Institute for All Competitive Exams
Practice Questions: Statement and Conclusions II

DIRECTIONS: In each question below is given a statement followed by two conclusions


numbered I and II. You have to assume everything in the statement to be true, then consider the
two conclusions together and decide which of them logically follows beyond a reasonable doubt
from the information given in the statement. Give answer:
(a) If only conclusion I follows.
(b) If only conclusion II follows.
(c) If either I or II follows.
(d) If neither I nor II follows, and
(e) If both I and II follow.

1. Statement:
An advertisement: ―Interest rate will be fixed on the basis of our bank's rate prevailing on the date
of deposit and re fixed every quarter thereafter‖
Conclusions:
I. It is left to the depositors to guard their interest.
II. The bank's interest rates are subject to change on a day-to-day basis depending on market
position.

2. Statement:
In order to enforce discipline on transport operators, the state government has decided to impose
a fine of Rs. 5,000 for the first excess tonne loaded in transport vehicle and Rs. 1,000 for each
subsequent tonne.
Conclusions:
I. People will follow some discipline when severe fine is imposed.
II. The state government has failed to understand the problem of transport operators.

3. Statement:
Researches have proved that people eating high fat diets coupled with decreased level of exercises
are prone to heart disease.
Conclusions:
I. People should reduce their high fat diet as a preventive method.
II. People must have sufficient level of exercise to reduce their chances of having heart disease.

4. Statement:
―Only those candidates with exceptional talent and strong motivation should apply.‖: An
advertisement.
Conclusions:
I. Candidates not fulfilling these criteria will not be considered.
II. It is possible to decide clearly who is talented and motivated.

5. Statement:
―Book your flat before 15 June and avail interest-free loan from the builders‖: an advertisement.
Conclusions:
I. No flat will be booked afterwards.
II. After 15 June, no loan will be provided.

6. Statement:
Medical advancement has increased life expectancy and also the expectation of all and sundry to
live longer and healthier but spiralling medical cost is a real spoil sport.
Conclusions:
Analytical Reasoning #3094, Sector 37D, Chandigarh. E-Mail: info@gjtutorial.com Page 53
Institute for All Competitive Exams
I. Advancement in medical science caters to the need of moneyed class alone.
II. Now-a-days medical profession has become a means of amassing wealth at the cost of ethics.

7. Statement:
Because of the portrayal of 'modern' women in films and private Indian channels which are
popular in Pakistan, Pakistanis believe that Indian women are totally under the western influence.
Conclusions:
I. Films and visual media largely depict the culture of a country.
II. Women in Pakistan are still uninfluenced by the western culture.

8. Statement:
The Chief Minister of State A expressed serious concern over the 32 percent dropout rate in
government schools.
Conclusions:
I. There is a growing concern in State A over the flight of students from schools.
II. State A is trying all out to increase its literacy rate.

9. Statement:
It is time for the bureaucrats to mould their approach from negative energy to positive energy.
Conclusions:
I. Bureaucracy is mixed in red-tapism and empire-building.
II. As a permanent executive of modern democracies, bureaucrats can change the dynamics of our
society.

10. Statement:
Declaration of strike by teachers at the time of paper evaluation has become an incurable disease
and a great threat to the government.
Conclusions:
I. Paper evaluation is the most important business of teachers.
II. Teachers are always in search of right opportunity to get their demands fulfilled.

11. Statement:
The ―Esteem‖ is one of the most fuel-efficient cars in its segment with a light foot ensuring 12.5
Km. to a litre in city without AC and about 11 Km. with AC on.
Conclusions:
I. AC in car is run by the same fuel that ensures the car's mobility.
II. With AC on, ―Fiat‖ ensures only 10 Km. to a litre.

12. Statement:
None of the bank unions could comprehend that the information technology wave was
unstoppable and unless banks learn to swim along with the tide, they would be swept away.
Conclusions:
I. There shall come a time when banks will be rendered redundant by the advent of information
technology.
II. Information technology is yet to make its impact on the banks.

13. Statement:
The Indian cement industry has witnessed strong consumption growth during 1999 wherein
cement consumption grew by 17.5 percent in April-Dec 1999 over the level in the same period
1998.
Conclusions:
I. There has been a marked growth in civil construction in 1999.
II. Repair works have been carried out on a massive scale in 1999.
Analytical Reasoning #3094, Sector 37D, Chandigarh. E-Mail: info@gjtutorial.com Page 54
Institute for All Competitive Exams
14. Statement:
The union finance minister in his latest budget speech declared that expenditure growth will be
curbed and structural changes in the composition of expenditure brought about.
Conclusions:
I. Expenditure has begun to take its toll on India's growth.
II. More than the amount of expenditure, it is composition that has affected the country's growth.

15. Statement:
If you are a consumer, chances are that you have run into problems with a product that you bought
or a service that was rendered.
Conclusions:
I. Producers do not run into problems with the products they manufacture.
II. Products and services are not defect free.

16. Statement:
The workplace of today and certainly tomorrow, is no longer going to remain an exclusive man's
world. 'It is going to transform itself into a much fairer person's world', where men and women
work together as individuals.
Conclusions:
I. Sexual bias towards men at workplace is being removed by and by.
II. Women are as efficient as their male counterpart.

17. Statement:
In the Union budget 2000-2001, defence allocation has been raised to Rs. 58,587 crore from Rs.
45,694 crore.
Conclusions:
I. Govt. is paying greater attention to national security.
II. The difference in budget allocation is due to inflationary reasons.

18. Statement:
If India aims at becoming an information technology (IT) superpower over the next decade, it is
time to acknowledge the need for producing a pool of professionals.
Conclusions:
I. India is on the verge of becoming IT superpower.
II. India has adequate IT professionals but in order to retain them here, there is need to reward
them suitably to prevent their migration abroad.

19. Statement:
In India, if one can grease the palm of a policeman, one can violate any law and rule, be it on traffic
or on chasing animals for sport or profit.
Conclusions:
I. Indian police is corrupt.
II. Chasing animal for sport or profit is considered as violation of laws in India.

20. Statement:
The Public Distribution System in recent years has been characterised by growing food stocks, a
low off take and an increasing economic cost of stocking grain.
Conclusions:
I. There has been sharp rise in production of grains in recent years.
II. PDS is urgently in need of reform.

Analytical Reasoning #3094, Sector 37D, Chandigarh. E-Mail: info@gjtutorial.com Page 55


Institute for All Competitive Exams
21. Statement:
―The maximum number of vacancies for the clerical cadre is 40, which will be filled through this
recruitment round‖: An advertisement of company 'A'.
Conclusions:
I. The company 'A' may appoint less than 40 clerks in this round.
II. The company 'A' may appoint 40 clerks in this round.

22. Statement:
Global ecological issues have eclipsed local environmental problems, which are being faced by the
poor societies.
Conclusions:
I. Poor societies always have to suffer because of their poverty.
II. Global ecological issues are not so important. Rich societies can bear with it.

23. Statement:
People in metropolitan city X have welcomed the recent Supreme Court order banning the
registration of private vehicles that do not conform to Euro II emission norms with immediate
effect for metropolitan city Y only.
Conclusions:
I. City 'X' has quite lower level of vehicular pollution than city Y.
II. Public vehicles do not contribute to the vehicular pollution.

24. Statement:
70% of the world's data is processed on XYZ company's platforms':An advertisement of XYZ:a
computer manufacturing company.
Conclusions:
I. There is no other company in the world which produces platforms of data processing.
II. Company XYZ has to make more efforts to market its platforms.

25. Statement:
The government of State 'A' has sought waiver of outstanding central loans of Rs. 4,000 crore and
a moratorium on repayment pending the waiver.
Conclusions:
I. Unlike other States, State A has no desire to make repayment of its loans.
State 'A's financial condition does not appear to be satisfactory.

Answer Key: Statement and Conclusions II

1. (b) 2. (a) 3. (e) 4. (e) 5. (d)


6. (d) 7. (e) 8. (a) 9. (d) 10. (b)
11. (a) 12. (d) 13. (a) 14. (a) 15. (b)
16. (a) 17. (c) 18. (a) 19. (e) 20. (e)
21. (c) 22. (d) 23. (d) 24. (d) 25. (b)

Analytical Reasoning #3094, Sector 37D, Chandigarh. E-Mail: info@gjtutorial.com Page 56


Institute for All Competitive Exams
Statement and Course of Action

This type 0f questions are designed to test candidate's decision making ability. Course of action is a
decision which a person makes invariably in his life following a problem. In these questions, a
problem is given in the form of statement and is followed by two or more decisions in the form of
course of action.
To solve this type of questions, students are advised to keep in mind following points:
 Correct course of action should either lessen the problem or improve the situation created by
the problem.
 Simple problem must have simple course of action and not a complex one which may create
more problems than to solve or reduce it.
 Course of action should be feasible and should relate with the practical aspect of life.
 A clear understanding about the type of question and method of solution can be developed by
way of solving a variety of questions given in this chapter subsequently.

Examples:

Directions: In each question below is given a statement followed by two courses of action
numbered I and II. A course of action is a step or administrative decision to be taken for
improvement, follow up or further action in regard to the problem policy etc. On the basis of the
information given in the statement, you have to assume everything in the statement to be true, and
then decide which of the two given courses of action logically follow. Give your answer
(a) If only I follows.
(b) If only II follows.
(c) If either I or II follows.
(d) If neither I nor II
(e) if both I and II follow.

1. Statement:
The Indian electronic component industry venturing into the West European markets faces tough
competition from the Japanese.
Courses of Action:
I. India should search for other international markets for its products.
II. India should improve the quality of the electronic components to compete with the Japanese in
capturing these markets.
Solution:
Escaping from the competition is not the right course of action. Step towards improving the quality
of the electronic components is the right course of action. Hence, (b) is the answer.

2. Statement:
A recent study shows that children below five die in the cities of the developing countries mainly
from diarrhoea and parasitic intestinal worms.
Courses of Action:
I. Govt, of the developing countries should take adequate measures to improve the hygienic
conditions in the cities.
II. Children below five years in the cities of the developing countries need to be kept under
constant medication.
Solution:
I. If Govt, concentrate to improve the hygienic condition in the cities, it may definitely reduce the
impact of problem. And this course of action appears to be feasible also. Hence, this course of
action follows.

Analytical Reasoning #3094, Sector 37D, Chandigarh. E-Mail: info@gjtutorial.com Page 57


Institute for All Competitive Exams
II. A constant medication is another practical feasible step, which would help minimising the cases
of death due to diarrhoea and intestinal worms. Therefore, course of action II also follows. Hence,
cur answer is (e).

3. Statement:
The officer-in-charge of a company had a hunch that some money was missing from the safe.
Courses of Action:
I. He should get it recounted with the help of the staff and check it with the balance sheet.
II. He should inform the police.
Solution:
Here a hunch about the money being lost does not provide any ground for the incident having
actually taken place, and hence action to inform the police will not lessen the problem but is more
likely to aggravate the situation.
A prudent step is to recount the money and tally with the books of account. Hence, course of action
I fits the situation. Therefore, (a) is our answer.

4. Statement:
Many cases of cholera were reported from a nearby village.
Courses of Action:
I. The question should be raised in the Legislative Assembly:
II. A team of doctors should be rushed to the village.
Solution:
Discussion of the problem in the Legislative Assembly is feasible but does not provide any
immediate relief to the victims. Hence, I is not the right course of action to be chosen for the
problem. However, immediate medical aid will definitely mitigate the problem to large extent.
Hence, course of action II is the remedial step. Therefore, (b) is our answer.

5. Statement:
Researchers are feeling agitated as libraries are not equipped to provide the right information to
the right users at the right time in the required format. Even the users are not aware about the
various services available for them.
Courses of Action:
I. All the information available in the libraries should be computerised to provide faster services to
the users.
II. Library staff should be trained in computer operations.
Solution:
Computerisation may definitely help the users to take best available information available in the
library. And training to the staff in computer can further add to betterment of services to the users.
Hence, both are feasible and therefore right courses of action. Hence, (e) is the answer.

6. Statement:
Youngsters are often found staring at obscene posters.
Courses of Action:
I. Children should be punished and penalised if they are found doing so.
II. Any display of such material should be banned.
Solution:
Punishment to the children is not a judicious step in the direction of dealing such problem.
However, putting a complete ban can help to solve the problem by way of preventing children to
get such exposure. Hence, (b) is our answer.

Analytical Reasoning #3094, Sector 37D, Chandigarh. E-Mail: info@gjtutorial.com Page 58


Institute for All Competitive Exams
Practice Questions: Statement and Course of Action

1. Statement:
Air export volumes have increased substantially over the past decade causing backlogs and
difficulties for air cargo agents because of increased demand for space and service.
Courses of Action:
1. Airlines and air cargo agents should jointly work out a solution to combat the problem.
II. The reasons for the increase in the volume of air export should be found out.

2. Statement:
The world conference of "Education for All" took place in 1990. Members who attended conference
endorsed the frame work of action for meeting the basic learning needs of all children.
Courses of Action:
I. India should suitably implement the action points of this conference.
II. India should also immediately organise this type of conference.

3. Statement:
About 30% to 40% of children, who are enrolled, do not attend the school on any given day.
Courses of Action:
I. More schools should be started.
II. Reasons for this absenteeism should be found out.

4. Statement:
Although the Indian economy is still heavily dependent on agriculture, its share in global
agriculture trade is less than the share of agriculture exports to total exports.
Courses of Action:
I. Efforts should be made to increase our agricultural production.
II. The exports of non-agricultural commodities should be reduced.

5. Statement:
Huge amount of resources are required to develop tourist places in a country like India, which is
endowed with vast coastlines, rivers, forests, temples etc.
Courses of Action:
I. More tourist resorts along the coastal line only should be started.
II. The tourist potential of India should be exploited.

6. Statement:
The Govt, has decided not to provide financial support to voluntary organisations from next five
year plan and has communicated that all such organisations should raise funds to meet their
financial needs.
Courses of Action:
I. Voluntary organisations should collaborate with foreign agencies.
II. They should explore other sources of financial support.

7. Statement:
The experts group on Technical Education has stressed that computer education should be
provided to children from primary school itself. It should be implemented in urban and rural
schools simultaneously.
Courses of Action:
I. Govt should issue instructions to all schools for computer education.
II. At least one teacher of each school should be trained in computer operations for teaching
children.
Analytical Reasoning #3094, Sector 37D, Chandigarh. E-Mail: info@gjtutorial.com Page 59
Institute for All Competitive Exams
8. Statement: Employment opportunities in private sector industry declined from 4.50 million
on December, 1991 to 4.15 million in November, 1992 due to large number of 'sick' industries—A
report.
Courses of Action:
I. Govt should start new industries.
II. Unemployed should be given concessional loans.

9. Statement: Ministry of Tourism in its one of the reports revealed that due to recent social
disturbance in the country, the number of foreign tourists has decreased considerably, which
resulted in a financial loss of Rs. 100 crore.
Courses of Action:
I. Government should provide financial support to the tourism sector.
II. Foreign tourists should be informed to visit the country at their risk.

10. Statement: There are more than 200 villages in the hill area of Uttar Pradesh which are
severely damaged due to cyclone and it causes an extra burden of Rs. 200 crore on State Govt, for
relief and rehabilitation work.
Courses of Action:
I. People of hill area should be shifted to other safer places.
II. State Govt should ask for more financial support from Central Government.

11. Statement:
If the retired professors of the same institutes are also invited to deliberate on restructuring of the
organisation, their contribution may be beneficial to the institute.
Courses of Action:
I. Management may seek opinion of the employees before calling retired professors.
II. Management should involve experienced people for the systematic restructuring of the
organisation.

12. Statement:
Experts Committee on Industrial Relations has revealed in their recent reports that not only the
internal problems of the organisations were successfully managed through quality circles but also
employees' motivation level was increased significantly.
Courses of Action:
I. Organisation should organise orientation programmes for their employees on quality circles.
II. Employees should be encouraged to join quality circle programmes.

13. Statement:
The Minister said the teachers are still not familiarised with need, importance and meaning of
population education in the higher education system. They are not even clearly aware about their
role and responsibilities in the population education programme.
Courses of Action:
I. Population education programme should be included in the college curriculum.
II. Orientation programme should be conducted for teachers on population education.

14. Statement: Researchers are feeling agitated as libraries are not equipped to provide the right
information to the right users at the right time in the required format. Even the users are not
aware about the various services available for them.
Courses of Action:
I. All the information available in the libraries should be computerised to provide faster services to
the users.
II. Library staff should be trained in computer operations.

Analytical Reasoning #3094, Sector 37D, Chandigarh. E-Mail: info@gjtutorial.com Page 60


Institute for All Competitive Exams
15. Statement:
The Chairman stressed the need for making education system more flexible and regretted that the
curriculum has not been revised in keeping with the pace of the changes taking place.
Courses of Action:
I. Curriculum should be reviewed and revised periodically.
II. System of education should be made more flexible.

16. Statement:
Reports of steep and continued decline in the inflows into the Gobindsagar reservoir of the Bhakra
Dam, coupled with a depleted stock of steam coal with the thermal power plants in the north, may
lead to a serious power crisis in the region.
Courses of Action:
I. The supply of steam coal to the thermal power plants needs to be immediately stepped up by the
Govt.
II. The Govt, should set up hydraulic power plants on other rivers in the region.

17. Statement:
Financial stringency prevented the Manipur Govt, from paying salaries to its employees since April
this year.
Courses of Action:
I. The Manipur Govt, should immediately curtail the staff strength at least by 30%.
II. The Manipur Govt, should reduce wasteful expenditure and arrange to pay the salaries of its
employees.

18. Statement:
The police department has come under a cloud with recent revelations that at least two senior
police officials are suspected to have been involved in the illegal sale of a large quantity of weapons
from the State police armoury.
Courses of Action:
I. A thorough investigation should be ordered by the State Govt, to bring out all those who are
involved into the illegal sale of arms.
II. State police armoury should be kept under Central Govt's control.

19. Statement:
The Indian electronic component venturing into the West European markets faces tough
competition from the Japanese.
Courses of Action:
I. India should search for other international market for its product.
II. India should improve the quality of the electronic components to compete with the Japanese in
capturing these markets.

20. Statement:
Certain mining industries in Gujarat may come to a standstill because of the notification issued by
the Department of Environment and Forest, banning mining operations and industries alike
within 25 kms of national park, the game sanctuary and reserve forest areas.
Courses of Action:
I. The department should be asked to immediately withdraw the notification.
II. The Govt, should make efforts to shift the parks, sanctuaries and reserve forests to other non-
mining areas.

Analytical Reasoning #3094, Sector 37D, Chandigarh. E-Mail: info@gjtutorial.com Page 61


Institute for All Competitive Exams
21. Statement:
The state Govt, has decided to declare "Kala Azar" as a notifiable disease under the Epidemic Act.
Under the Epidemic Act, 1897, family members or neighbours of the patient are liable to be
punished in case they did not inform the State authorities.
Courses of Action:
I. Efforts should be made to efficiently implement the Act.
II. The cases of punishment should be propagated through mass media so that more people
become aware of the stern action.

22. Statement:
Every year, at the beginning or at the end of the monsoon, we have some cases of conjunctivitis,
but this year, it seems to be a major epidemic, witnessed after nearly four years.
Courses of Action:
I. Precautionary measures should be taken after every four years to check the epidemic.
II. People should be advised to drink boiled water during monsoon season.

23. Statement:
The committee has criticised the institute for its failure to implement a dozen of regular
programmes despite an increase in the staff strength and not drawing up a firm action plan for
studies and researches.
Courses of Action:
I. The broad objectives of the institute should be redefined to implement a practical action plan.
II. The "Institute should give a report on reasons for not having implemented the planned
programmes.

24. Statement:
The rates of interest on Post Office recurring deposit accounts have been increased with effect from
Oct. 1. This has been done to attract more deposits.
Courses of Action:
I. Efforts should also be made to make the public aware about this increase in the rate of interest.
II. If the deposits do not increase in next six months, the rate of interest should be further
increased.

25. Statement:
The army has been alerted in the district following floods triggered by incessant rains.
Courses of Action:
I. Relief to flood affected people should be arranged.
II. Supply of food articles should be arranged.
III. Adequate medical facilities should be arranged,
(a) Only I follows
(b) Only II follows
(c) Only I and III follow
(d) None
(e) All follow

26. Statement:
Faced with a serious resource crunch and a depressing overall economic scenario, Orissa is
unlikely to achieve the targeted percent compound annual growth rate during the 8th plan.
Courses of Action:
I. The target growth should be reduced for the next Plan. II. The reasons for the failures should be
studied.
III. Orissa's performance should be compared with that of other States.

Analytical Reasoning #3094, Sector 37D, Chandigarh. E-Mail: info@gjtutorial.com Page 62


Institute for All Competitive Exams
(a) Only I follows
(b) Only I and III follow
(c)Only II and III follow
(d) None follows
(e) All follow

27. Statement:
Over 27,000 bonded labourers identified and freed are still awaiting rehabilitation.
Courses of Action:
I. More cases of bonded labourers should be identified.
II. Till the proper rehabilitation facilities are available, the bonded labourers should not be freed.
III. The impediments in the way of speedy and proper rehabilitation of bonded labourers should be
removed,
(a) Only I follows
(b) Only II follows
(c) Only III follows
(d) Only II and III follow
(e) None follows

28. Statement:
Higher disposal costs encourage those who produce waste to look for cheaper ways to get rid of it.
Courses of Action:
I. The disposal costs should be made higher. II. The disposal costs should be brought down.
III. A committee should be set up to study the details in this respect.
(a) Only I follows
(b) Only II follows
(c) Either I or II follows
(d) Only II and III follow
(e) All follow

29. Statement:
If the faculty members also join the strike, there is going to be a serious problem.
Courses of Action:
I. The faculty members should be persuaded not to go on strike.
II. Those faculty members who join the strike should be suspended.
III. The management should not worry about such small things.
(a) Only I follows
(b) Only I and II follow
(c) Only II and III follow
(d) None follows
(e) All follow

30. Statement:
According to the officials, paucity of funds with the organisation has led to the pathetic condition
of this brilliant architectural structure.
Courses of Action:
I. A new architectural structure for the building should be designed.
II. The reasons for the poor condition of the structure should be found out.
III. Grant should be given to improve the condition of the structure.
(a) Only I follows
(b) Only II follows
(c) Only III follows
(d) Only II and III follow
Analytical Reasoning #3094, Sector 37D, Chandigarh. E-Mail: info@gjtutorial.com Page 63
Institute for All Competitive Exams
(e) Only I and III follow

31. Statement: In the Teacher's Day function, Shri Sharma, a State awardee and a retired
Principal, had questioned the celebration of Teacher's Day in today's materialistic world.
Courses of Action:
I. The expenditure of Teacher's Day celebration should be reduced.
II. More funds should be allocated for the celebration of Teacher's Day.
III. The role and responsibilities of teachers should be seen in today's perspective.
(a) All follow
(b) None follows
(c) Either I or II follows
(d) Only III follows
(e) Only II and III follow

32. Statement:
In the city, over 75 percent of the people are-living in slums and sub-standard houses, which is a
reflection on the housing and urban development policies of the Government.
Courses of Action:
I. There should be a separate department looking after housing and urban development.
II. The policies in regard to urban housing should be reviewed.
III. The policies regarding rural housing should also be reviewed so that such problems could be
avoided in rural areas.
(a) Only I follows
(b) Only II follows
(c) Only II and III follow
(d) Either II or III follows
(e) Only I and II follow

33. Statement:
Lack of coordination between the university, its colleges and various authorities has resulted in
students ousted from one college seeking migration to another.
Courses of Action:
I. If a student is ousted from a college, the information should be sent to all the other colleges of
university.
II. The admissions to all the colleges of the universities should be handled by the university
directly.
III. A separate section should be made for taking strict action against students indulging in anti-
social activities,
(a) Only I follows
(b) Only II follows
(c) Only III follows
(d) Only I and III follow
(e) Only II and III follow

34. Statement:
The Institute has fixed for then investors a validity period of one year for transfer forms for some
of its listed schemes.
Courses of Action:
I. The Institute should consult investors before fixing the duration of validity period.
II. The investors should be duly informed about the validity period.
III. List of schemes covered under this validity period should be communicated.
(a) All follow
(b) Only I and II follow
Analytical Reasoning #3094, Sector 37D, Chandigarh. E-Mail: info@gjtutorial.com Page 64
Institute for All Competitive Exams
(c) Only I and III follow
(d) Only III follows
(e) Only II and III follow

35. Statement:
Without the active cooperation between the proprietor and the employees of the mill, it cannot
remain a profitable concern for long.
Courses of Action:
I. The mill should be closed down.
II. The workers should be asked to cooperate with the owners.
III. The owner should be asked to cooperate with the employees.
(a) Only I and II follow
(b) None follows
(c) All follow
(d) Only I and III follow
(e) None of these

36. Statement:
Some strains of mosquito have become resistant to chloroquine, the widely used medicine for
malaria patients.
Courses of Action:
I. Selling of chloroquine should be stopped.
II. Researchers should develop a new medicine for patients affected by such mosquitoes.
III. All the patients suffering from malaria should be checked for identification of casual mosquito,
(a) None follows
(b) Only I and II follow
(c) Only II and III follow
(d) All follow
(e) None of these

37. Statement:
Incessant rain for the past several days has posed the problem of overflowing and flood as the river
bed is full of silt and mud.
Courses of Action:
I. The people residing near the rivers should be shifted to a safe place.
II. The people should be made aware about the immediate danger over radio/television.
III. The silt and mud from the river bed should be cleared immediately after the receding of the
water level.
(a) Only I and II follow
(b) Only II and III follow
(c) None of these
(d) All follow
(e) Only I and III follow

38. Statement:
Every year thousands of eligible students do not get admission in colleges, both in urban and rural
areas, after passing their school leaving certificate exams.
Courses of Action:
I. More colleges should be set up in both urban and rural areas.
II. The number of schools in both urban and rural areas should be reduced.
III. No school should offer vocational courses to equip students for taking up their vocation after
completing their school education.

Analytical Reasoning #3094, Sector 37D, Chandigarh. E-Mail: info@gjtutorial.com Page 65


Institute for All Competitive Exams
(a) Only I follows
(b) Only II and III follow
(c) All follow
(d) Only I and III follow
(e) None of these

39. Statement:
Any further increase in pollution level in the city, by way of industrial effluent and automobile
exhaustion, would pose a severe threat to the inhabitants.
Courses of Action:
I. All the factories in the city should be immediately closed down.
II. The automobiles should not be allowed to ply on the road for more than four hours a day.
III. The Government should restrict the issue of fresh licenses to factories and automobiles.
(a) Only III follows
(b) All follow
(c) Only II follows
(d) None follows
(e) None of these

40. Statement:
India's pre-eminent position in the world black pepper production and trade is in danger as some
of the countries, which recently started production of the "king of the spices" crop from Indian root
stocks, are farming better by adopting modern cultivation practices.
Courses of Action:
I. India should immediately stop supplying root stocks of black pepper to other countries.
II. India should adopt modern technology for cultivating black pepper to compete in the
international market.
III. India should reduce the price of its black pepper to remain competitive in the world market,
(a) All follow
(b) Only II follows
(c) Only I follows
(d) Only III follows
(e) None of these

Analytical Reasoning #3094, Sector 37D, Chandigarh. E-Mail: info@gjtutorial.com Page 66


Institute for All Competitive Exams
Descriptive Answers (Statement and Course of Action)

1 (a):
The problem faced by the airlines and cargo agents is the non-availability of cargo space.
Therefore, right course of action is to combat the problem.
2 (a):
Implementation of such action plans, as framed by the conference, is a right course of action as it
will give an immediate and effective impact on the standard of education for the children.
3 (b):
Reasons for this absenteeism should be found out as it will help to check such absenteeism of
students from the class.
4 (a):
Only by increasing our agricultural production, we can have a better position in international
agricultural trade. Reduction in non-agricultural commodities will further worsen our position. So,
only I is the right course of action.
5 (b):
As per statement, India is already endowed with tourist spots. Therefore, these existing potential
should be exploited.
6 (b):
Obviously, if Govt has decided to stop financial support to voluntary organisations, these
organisations should find other source of financial support. Hence, (II) is the right course of
action.
7 (e):
Both are the right courses of action for effective implementation of computer education at primary
level in urban and rural schools.
8 (e):
Both are the right courses of action as each of the courses of action brightens the chance of
employment opportunity.
9 (d):
Financial support to the tourism sector is not a permanent solution. Secondly, constant risk for the
foreign tourists will never encourage tourism in
India. Hence, neither of the actions is the right solution in this direction.
10 (e):
Victims should be shifted to the safer place. Secondly, affected State should ask for more financial
help from Central Govt, to accelerate the relief operation.
11 (b):
Valuable suggestions from the retired professors will be helpful for restructuring of organisation.
Hence, management should involve such experienced people. Seeking permission from the
employee is not the right course of action.
12 (e):
Statement emphasises the benefits and effectiveness of quality circle. Hence, it is advisable that
quality circle should be encouraged in the organisation and at the same time employees should be
persuaded to join it. Hence, both the actions are right courses of action.
13 (b):
Statement points out the unacquaintance of the teachers with population education, hence
orientation programme should be conducted for teachers. Hence, action (II) is the right course of
action.
14 (e):
Statement points out the ill-managed working of libraries with regard to providing right
information. Computerisation will help to organise the information systematically. Secondly,
library staff should be trained to make full and correct use of computers. Hence, both the actions
are right courses of action.
Analytical Reasoning #3094, Sector 37D, Chandigarh. E-Mail: info@gjtutorial.com Page 67
Institute for All Competitive Exams
15 (e):
Statement speaks of inadequacies of the education system and emphasises the need for flexibility
and revision. Hence, both the actions are right courses of action to update the curriculum.
16 (a):
Statement points out the crisis faced by the power plant and one of the crises is depletion of stock
of steam coal with the thermal power plants in the north. Therefore, it is the right course of action
to supply the steam coal to the thermal power plants. Secondly, since the water inflows into the
Gobindsagar have declined, setting up of another hydraulic power plant will be futile.
17 (b):
Curtailing wasteful expenditure is the right step towards making provision for money to be
distributed against pending salaries. Curtailing staff-strength is not the right solution. Hence,
action (II) is the right course of action.
18 (a):
Keeping the State police armoury under the supervision of Central Govt, control is not right course
of action to bring the guilty to the book. However, an investigation may help to reveal those
involved into the illegal acts, hence, only action (I) is the right course of action.
19 (b):
Escaping from the problem is not the right way to tackle the problem. Therefore, instead of finding
other international market, India should improve the quality of the electronic components.
Therefore, action (II) is the right course of action.
20 (d):
Action (I) is not the right course because action by the Govt, to issue notification has been taken in
view of the threat to the environment. Shifting parks, sanctuaries and reserve forests to other place
is not feasible. Hence, action (II) is also not a right course of action.
21 (e):
It is given in the statement that 'Kala Azar' has been declared as notifiable disease. Since it is a
good effort by the Govt., hence effort should be made to implement the Act. Moreover, a deterrent
in the form of punishment will ensure effective implementation of the Act.
22 (b):
It is not necessary that such an epidemic occurs after four years. Hence, action (I) is not the right
course of action. Secondly, prevention during monsoon season is the right step to face the problem
of conjunctivitis. So, action (II) is the right course of action.
23 (e):
The efficiency of the institute can be boosted up by redefining the institute‘s objectives laid down
for this purpose and by way of finding the reasons for the failure. Hence, both the actions are right
courses.
24 (e):
It is given in the statement that rate of interest has been increased to attract more deposits. Hence,
public should be made aware of such increase so that both the parties should be benefitted. If the
response for the deposits is lukewarm, the rate of interest should be further increased. Hence, both
the actions are right courses of action.
25 (e):
All the courses of action follow for pursuing the problem as defined in the statement, because flood
affected persons need all sort of relief, i.e., food, medical facilities, etc.
26 (c):
Action II and III follow for pursuing the problem as defined in the statement. Reasons for failure
should be studied and performance of the affected State should be compared with that of other
States.
27 (c):
Only HI action plan provides a feasible and effective course to combat the problem, that
impediments in the way of speedy and proper rehabilitation should be removed. Action (I) and (II)
are not effective courses.
28 (d):
Analytical Reasoning #3094, Sector 37D, Chandigarh. E-Mail: info@gjtutorial.com Page 68
Institute for All Competitive Exams
Action II and III are feasible and effective to combat the problem.
29 (a):
The best way to prevent faculty members to go on strike is to persuade them. Hence, action (I)
shows the right course of action.
30 (c):
Condition of the architectural structure can be improved by way of adequate finance, hence action
(III), that grant should be given to improve the condition of the structure, is the right course of
action.
31 (d):
In the statement, celebration of Teachers Day in today's materialistic world is in question which
means that the role and responsibilities of teachers should be seen in today's perspective. Hence,
action (III) is the right course of action.
32 (b):
The statement speaks of the failure of housing and urban development policies of the government;
hence the policies in regard to urban housing should be reviewed.
33 (a):
Only action (I) is the right course of action which says that if a student is ousted from a college, the
information should be sent to all the other colleges of university. This is the way by which
coordination between universities and colleges can be maintained.
34 (e):
The investors should be informed about the validity period, and list of schemes covered under this
validity period should be communicated so that investors could be benefitted of this facility.
35 (d):
It is given in the statement that cooperation between the proprietor and the employees is required
for a profitable reason. It is, therefore, required that both employees and proprietor should
cooperate each other.
36 (c):
It is given in the statement that chloroquine has become ineffective for malaria patients. Therefore,
researchers should develop a new medicine for the patients and all the patients suffering from
malaria should be checked for identification of causal mosquito.
37 (d):
All actions given are right course of action following problem caused by the incessant rain for the
past several days because each of the courses will provide relief to the victims.
38 (d):
The statement describes the problem of admission in colleges faced by students. Both the courses
of action (I) and (III) are the right step towards providing solution of this problem.
39 (a): Only action (III) is the right course of action because other courses are not feasible.
40 (b): Only better quality can put India back in the competitive field of black pepper production.
So, India should go for modern technology for cultivating black pepper to compete in the
international market.

Analytical Reasoning #3094, Sector 37D, Chandigarh. E-Mail: info@gjtutorial.com Page 69


Institute for All Competitive Exams
Practice Questions: Statement and Course of Action II

Directions: Herein each of the following questions a statement is followed by two courses of
action. A course of action is taken for improvement, follow-up, etc. Read the statement carefully
and give answers
1. If only course of action I follows.
2. If only course of action II follows.
3. If either course of action I or II follows.
4. If both course of action I and II follows.
5. If neither course of action I nor II follows.

1.
Statements:
Over a thousand units, employing more than two lakh people, in Mayapuri in West Delhi, are
facing acute power shortage.
Courses of action:
I. The industrial units in Mayapuri should be closed down immediately.
II. The government should arrange power for those units by cutting some of the supply to
residential areas.

2.
Statements:
The management of the institute is planning to deliberate on restructuring of the organisation.
Course of action: I.
Management should seek advice from the experienced employees.
II. Management should involve experienced people for the systematic restructuring of the
institute.

3.
Statements:
There occurred a major train accident at Gaisal for reason unknown
Courses of action:
I. An inquiry should be set up.
II. A harsher punishment should be given for carelessness.

4.
Statements:
Recently, there was a warlike situation between India and Pakistan, and even now, tension is high.
Courses of action:
I. Both countries should initiate talks on alleviating tension taking a mediator in confidence.
II. Both countries should initiate talks on alleviating tension without taking a mediator into
confidence.

5.
Statements:
August 11 was the day of the millennium‘s last solar eclipse, Seeing this with open eyes directly, as
scientists said, was very detrimental.
Courses of action:
I. The government should have made proper arrangement to make people aware of the danger.
II. There should have been a live telecast of the incident on television

Analytical Reasoning #3094, Sector 37D, Chandigarh. E-Mail: info@gjtutorial.com Page 70


Institute for All Competitive Exams
Directions: Herein each of the following questions a statement is followed by two course of
action. A course of action is taken for improvement, follow-up, etc. Read the statement carefully
and give answers
1. If only course of action I follows.
2. If only course of action II follows.
3. If either course of action I or II follows.
4. If both course of action I and II follows.
5. If neither course of action I nor II follows.

6.
Statements:
The financial performance of the Central public sector undertakings has touched a new low in
1998-99
Courses of action:
I. Each and every loss making public sector unit should be privatised.
II. Management upgradation and total cost management should be emphasised.

7.
Statements:
Women have been found to be involved in crimes such as kidnapping, rape, robbery, murder,
drug-pedalling and forcing young girls into prostitution
Courses of action:
I. The police have to put up notices in police stations all over the city to track them down.
II. The police should have separate teams to track down proclaimed offenders.

8.
Statements:
A rogue file disguised as an e-mail message from an acquaintance infects the nervous system of the
computer.
Courses of action:
I. The computer security experts should take antivirus precautions.
II. No operator should open an e-mail message with an attachment.

9.
Statement:
Indian system encourages corruption due to the scarcity of goods and services, red tapism and lack
of transparency and elaborate legal procedures.
Courses of action:
I. Government should bring in legislative reforms for more transparency in the dealings of
different government departments simplifying procedures.
II. Provision of effective punishment for corrupt government servants are to be introduced.

10.
Statement:
Unauthorised hawking, begging, vending of liquor, smoking in Railways are all prohibited by rules,
but nevertheless they are all carried on with impunity.
Courses of action:
I. Indian Railways Act should be amended to make smoking and other illegal activities in the train
an offence.
II. Those found breaking the law should be fined on the spot.

Analytical Reasoning #3094, Sector 37D, Chandigarh. E-Mail: info@gjtutorial.com Page 71


Institute for All Competitive Exams
Directions: Here in each of the following questions a statement is followed by two course of
action. A course of action is taken for improvement, follow-up, etc. Read the statement carefully
and give answers
1. If only course of action I follows
2. If only course of action II follows
3. If either course of action I or II follows
4. If both course of action I and II follows
5. If neither course of action I nor II follows

11.
Statements:
Mr. Manohar Jalan, an employee of company X, shows no more interest in his work.
Courses of action:
I. The personnel officer should look into Mr. Jalan's problems and try to sort them out.
II. The company should depute Mr. Jalan to some other work for a period of time.

12.
Statements:
The big cities have become gaping sores on the Earth's environment.
Courses of action:
I. Public investment should be made into sewage treatment and garbage disposal to protect
environment of big cities.
II. Their control should be transferred from the state government to the Central government.

13.
Statements:
The Indian Constitution is no more suitable for the needs of the country.
Courses of action:
I. A review of the Constitution should be taken up.
II. The Constitution should be thrown away.

14.
Statements:
The exodus from villages to cities is detrimental to both.
Courses of action:
I. Rural postings should be made mandatory for government servants as well as professionals.
II. Employment generation schemes should be launched in rural areas.

15.
Statements:
Nuclear power can not make a country secure.
Courses of action:
I. We must stop further expenses on increasing our nuclear power.
II. We must destroy our nuclear capability.

Answer Key (Statement and Course of Action II)


1: 24434 6: 22112 11: 41141

Analytical Reasoning #3094, Sector 37D, Chandigarh. E-Mail: info@gjtutorial.com Page 72


Institute for All Competitive Exams
Argument Strengthen/ Weaken

Directions: Each interrogative statement is followed by two arguments. You are to pick up one of
the following responses as the answer.

1. Should illiterate be debarred from voting?


(x) Yes, because illiterate people do not have political consciousness.
(y) No, because democracy presupposes political equality among all.
1. Argument ‗x‘ is forceful
2. Argument ‗y‘ is forceful
3. Neither ‗x‘ nor ‗y‘ is forceful
4. Both ‗x‘ and ‗y‘ are forceful
5. Both ‗x‘ and ‗y‘ are irrelevant

2. Should strikes be banned in all services?


(x) Yes, because strikes cause a lot of inconvenience to the people.
(y) No, because this is the only method of getting grievances redressed.
1. Argument ‗x‘ is forceful
2. Argument ‗y‘ is forceful
3. Neither ‗x‘ nor ‗y‘ is forceful
4. Both ‗x‘ and ‗y‘ are forceful
5. Both ‗x‘ and ‗y‘ are irrelevant

3. Should capital punishment be abolished?


(x) Yes, because it is inhuman to kill people.
(y) No, because such a step will give rise to crimes.
1. Argument ‗x‘ is forceful
2. Argument ‗y‘ is forceful
3. Neither ‗x‘ nor ‗y‘ is forceful
4. Both ‗x‘ and ‗y‘ are forceful
5. Both ‗x‘ and ‗y‘ are irrelevant

Directions: Each interrogative statement is followed by two arguments M & N. You are to pick up
one of the following responses as the answer.

4. Should all the nationalised banks be amalgamated into one?


(M) Yes, because the administration of the banks will be uniform.
(N) No, because small units are managed better.
1. Only argument ‗M‘ is forceful
2. Only argument ‗N‘ is forceful
3. Both ‗M‘ and ‗N‘ are forceful
4. Neither ‗M‘ nor ‗N‘ are forceful
5. Either ‗M‘ or ‗N‘ are irrelevant

5. Should India have homes for the old?


(M) Yes, because in India also the young are becoming indifferent to the old.
(N) No, because the old in India will not like to move to those homes as they are attached to their
children.
1. Only ‗M‘ is forceful argument
2. Only ‗N‘ is forceful argument
3. Both ‗M‘ and ‗N‘ is forceful arguments
4. Neither ‗M‘ nor ‗N‘ are forceful
Analytical Reasoning #3094, Sector 37D, Chandigarh. E-Mail: info@gjtutorial.com Page 73
Institute for All Competitive Exams
5. Either ‗M‘ or ‗N‘ are irrelevant

6. Should cow slaughter be banned in India?


(M) Yes, because such a step would mean respecting the religious feeling of the majority
community i.e. Hindus.
(N) No, because dry and old cows would be a burden upon the society.
1. Only argument ‗M‘ is forceful
2. Only argument ‗N‘ is forceful
3. Both ‗M‘ and ‗N‘ is forceful
4. Neither ‗M‘ nor ‗N‘ are forceful
5. Either ‗M‘ or ‗N‘ are irrelevant

7. Should we have only two political parties in India ?


(M) Yes, because many political parties create confusion in the country.
(N) No, because large number of political parties will give a large number of political ideologies
from which voters can choose one.
1. Only argument ‗M‘ is forceful
2. Only argument ‗N‘ is forceful
3. Both ‗M‘ and ‗N‘ is forceful
4. Neither ‗M‘ nor ‗N‘ are forceful
5. Either ‗M‘ or ‗N‘ are irrelevant

8. Does the world need a new economic order?


(M) Yes, because the rich nations are exploiting the poor, politically and economically.
(N) No, because no economic order can remove economic disparities.
1. Only argument ‗M‘ is forceful
2. Only argument ‗N‘ is forceful
3. Both ‗M‘ and ‗N‘ is forceful
4. Neither ‗M‘ nor ‗N‘ are forceful
5. Either ‗M‘ or ‗N‘ are irrelevant

9. Is the policy of non-violence still a wise course?


(M) Yes, because violence will result in greater violence.
(N) No, because non-violence is the weapon of the weak.
1. Only argument ‗M‘ is forceful
2. Only argument ‗N‘ is forceful
3. Both ‗M‘ and ‗N‘ is forceful
4. Neither ‗M‘ nor ‗N‘ are forceful
5. Either ‗M‘ or ‗N‘ are irrelevant

10. Was it wise to hold Asian Games in India?


(M) Yes, because it enhanced the prestige of India in World community.
(N) No, because India had to spend a huge amount of money on these games.
1. Only argument ‗M‘ is forceful
2. Only argument ‗N‘ is forceful
3. Both ‗M‘ and ‗N‘ is forceful
4. Neither ‗M‘ nor ‗N‘ are forceful
5. Either ‗M‘ or ‗N‘ are irrelevant

Directions: Read the statement carefully presented in each question and the arguments
following. You are to find out which of the two arguments is/are strong. Mark answer.
1. If argument A is strong
2. If argument B is strong
Analytical Reasoning #3094, Sector 37D, Chandigarh. E-Mail: info@gjtutorial.com Page 74
Institute for All Competitive Exams
3. If both arguments are strong
4. If none of the arguments are strong and
5. If either argument I or II is strong

11.
Statements:
Should crime against women be punished by capital punishment?
Arguments:
I. No, capital punishment shouldn‘t be given to anybody unless he has killed someone.
II. Yes, that would reduce the incidence of such crimes.

12.
Statements:
Should professional blood donation be banned?
Arguments:
I. Yes, It leads to spread of diseases like AIDS.
II. No, the move would like professional blood donors jobless.

13.
Statements:
Should love marriages be encouraged by society?
Arguments:
I. Yes, it would eliminate the dowry system along with many other social evils.
II. No. statistics show that love marriages do not last as long as arranged marriages.

14.
Statements:
Should computer courses be made compulsory in schools?
Arguments:
I. Yes, training in computers from an early age would help students later on in their careers.
II. No, course in school are very demanding as it is and computer training would only add to the
burden on the children.

15.
Statements:
Should the government stop giving subsidy to loss making public sector units ?
Arguments:
I. Yes , subsidies can never cure the ailment in loss making public sector units.
II. No, public sector units have carved out a niche for themselves in India‘s developmental process
and they should be sustained at all costs.

Answer Key (Argument Strengthen / Weaken)


1: 21334 6: 11321 11: 31313

Analytical Reasoning #3094, Sector 37D, Chandigarh. E-Mail: info@gjtutorial.com Page 75


Institute for All Competitive Exams
Probably True & False

Directions:
Herein below is given a passage followed by several possible facts stated in the passage. You have
to examine each inference separately in the context of the passage and decide upon its degree of
truth or falsity. Mark answer
(1) if the inference is ―definitely true‖ i.e. it properly follows from the statement of facts given.
(2) if the inference is ―probably true‖ though not ―definitely‖ true' in the light of the facts given
(3) if the ―data are inadequate‖ i.e. from the facts given you cannot say whether the inference is
likely to be true or false.
(4) if the inference is ― probably false‖ though not ―definitely false‖ in the light of the facts given.
(5) if the inference is ― definitely false‖ i.e., it cannot possibly be drawn from the facts given.

PARAGRAPH I
In a country where at least ten percent of the population suffers from some handicap or the other,
it is surprising that people are not sufficiently sensitive to the needs of the handicapped. The
government‘s nonchalance too has contributed to the failure of the mainstream education system
that has not risen to the special demands of those with learning, physical or mental difficulties.
There is a dearth of reading material for them and in most ―special needs‖ school teachers have to
constantly struggle to improvise standard textbooks to make them suitable for slow learners, says
Ms. Vinita Krishnan of ―Tamanna‖ an organisation which works with such children.
1. We should be sympathetic to the handicapped peoples‘ cause.
2. The government should devise some alternative education system for the handicapped.
3. In India, teachers in handicapped schools should improvise standard textbooks to make them
adjustable to the needs of the pupils.
4. The government should spend more money for the development of the handicapped.
5. Handicapped people have lower absorption capacity than a normal individual.

PARAGRAPH II
Urban services have not expanded fast enough to cope with urban expansion. Low investment
allocations have tended to be underspent. Both public (e.g. water and sewerage) and private (e.g.
low-income area housing) infrastructure quality has declined. The impact of the environment in
which children live and the supporting services available to them when they fall ill seems clear. The
decline in average food availability and the rise in absolute poverty point in the same
unsatisfactory direction.
6. There is nothing to boast about urban services.
7. The public transport system is in the hands of private sector.
8. Low-cost urban housing is one of the priorities.
9. The environment around plays an important role on the health status.
10. Though adequate provisions of funds were made but they remained unspent.

PARAGRAPH III
The historical experience of how country after country progressed from poverty to prosperity over
the last two hundred years suggests that the transformation in each case was generally driven by
one sector, which became the engine of the economy. In Britain it was textiles; in the US the
industrial revolution was led by the railways. In today‘s language, that nation‘s competitive
advantage emanates from its leading sector. Over the last fifty years India has struggled
unsuccessfully to find its lead sector or its source of competitive advantage, and this explains, in
part, its inability to create an industrial revolution. Now, it may have found it in the information or
knowledge sectors of the economy.
11. US and Britain are the most prosperous countries in the world
12. India hundred years ago, all countries of the world were poor.
Analytical Reasoning #3094, Sector 37D, Chandigarh. E-Mail: info@gjtutorial.com Page 76
Institute for All Competitive Exams
13. Two hundred years ago, all countries of the world were poor
14. In near future India will not be listed among prosperous countries.
15. Indians are the most talented in information technology

PARAGRAPH IV
The development model we chose and implemented, it is true, was not the one that the Mahatma
and the Gandhian economists like Dr JC Kumarappa had in mind. The assassination of the
Mahatma immediately after Independence brought about a sea change in the ideological
atmosphere of the country. Gandhiji‘s name was invoked in a ritual manner even on occasions
when patently un-Gandhian and even anti-Gandhian programmes and projects were launched.
This has become almost a national habit with us.
16. JC Kumarappa was a development economist.
17. Mahatma Gandhi was assassinated in 1948.
18. Gandhi‘s ideals were given a go-by soon after his assassination.
19. Economists have cheated the nation in the name of Gandhi.
20. The author of the passage favours the Gandhian model of development.

Answer Key (Probably True & False)


1: 22121 6: 23115 11: 22143 16: 23122

Analytical Reasoning #3094, Sector 37D, Chandigarh. E-Mail: info@gjtutorial.com Page 77


Institute for All Competitive Exams
Conditional Selection

Directions: Study the following information to answer the given questions. Following are the
criteria to give preferences in the allotment of berths to the wait-listed old citizens by the Indian
Railways.
The person who are wait-listed must:
1. Be more than 60 years of age.
2. Have obtained at least ‗C‘ ratings in no. of hours of travelling and no. of times of journey on
Indian Railways during the last five years. For this five ratings –A, B, C, D and E – have been
created on the basis of ―Railways travelling report‖ with the lowest ratings of A and highest rating
of E.
3. Have obtained at least ‗D‘ rating in total distance travelled and no. of trains used for his/her
journey by Indian Railways during the last three years. For this six ratings A, B, C, D, E and F have
been formulated on the basis of ―Railways travelling report‖ with lowest rating of A and the highest
rating of F.
4. Have retired from defence services.
However in case a candidate who fulfils all other criteria EXCEPT.
(A) ―4‖ above, but has retired from other government departments or PSUs, may be referred to
Chief Reservation Clerk.
(B) ―1‖ above, but be more than 58 years of age and has submitted a medical certificate in favour of
his/ her unfitness, may be referred to Medical Officer, IR.

Based on these criteria and information provided against each candidate, decide the course of
action. You are not to assume anything. If the data provided are not adequate to decide the given
course of action, your answer will be ―Data inadequate‖. Give answer:
1. If preferences to be given
2. If data are inadequate
3. If case is to be referred to Chief Reservation Clerk
4. If preferences not to be given
5. If case is to be referred to Medical Officer

1. S Padmanavan, a retired defence personnel of 61 years of age, has obtained ‗D‘ ratings in no. of
hours of travelling and no. of trains used for journey during the last five years in RTR. He has also
obtained ‗E‘ ratings in total distance travelled and no. of times of journey in total distance travelled
and no. of times of journey on Indian Railways during the last three years in RTR.

2. 70-year-old Abdul Aziz is the father of Nazir Ahmed, a defence personnel. He retired from SAIL,
a PSU, in 1990. He has obtained at least ‗E‘ ratings in all the four scales on the basis of RTR during
the last fifteen years. Because of his old age he is unfit and so has submitted a medical certificate.

3. 62-year-old Dada Bhai has obtained C, D, C and E ratings in no. of times of journey, no. of
trains used for his journey, no. of hours of travelling and total distance travelled in RTR for the last
five years respectively. He is a retired doctor of a public sector unit.

4. Sabita Roy is famous lawyer of her town. She is 59 years old. As she is a consultant in a Supreme
Court she visit regularly from her town Howrah to Delhi by Indian Railways. She has obtained E
ratings in all the categories for the last ten years in the RTR. This time she has submitted the
medical certificate for her unfitness.

5. Ram Lakhan Prasad is a retired school teacher. He has obtained E rating in no. of hours of
travelling and no. of times of journey on Indian Railways during the last five years in RTR. He has
obtained D, E, C and D ratings in last four years respectively in total distance travelled and no. of
Analytical Reasoning #3094, Sector 37D, Chandigarh. E-Mail: info@gjtutorial.com Page 78
Institute for All Competitive Exams
trains used for his journey in RTR. He retired in 1998 at the age of 60 years, from Kendriya
Vidyalaya, a government school.

6. Om Prakash Rai is an ex-serviceman and now runs his dairy in his village, 63-year-old Rai has
obtained F ratings in total distance travelled and no. of trains used for his journey during the last
four years if RTR. He has obtained D, E, C, B and E ratings in no. of hours of travelling and no. of
times of journey in last five years respectively in RTR.

7. Mahesh Jha has been given best ratings in all the categories for the last ten years in RTR by
Indian railways. He is a 65-year-old retired engineer from NTPC, a PSU.

Directions: Read the following information carefully an answer the questions given below it.
The candidate must be
1. B.Com., with minimum 65% marks.
2. CAIIB with minimum 50% marks.
3. Having working experience of minimum one year in foreign exchange department of any bank.
4. Age between 24 years and 30 years as on 1.7.1999.
5. Ready to keep a deposit of Rs. 25, 000 with the bank.

However, if the candidate fulfils all other criteria except


(A) ―2‖ above, but has completed CAIIB part i with minimum 60% marks his case may be referred
to the general manager.
(B) ―3‖ above, but has 4 years experience of working in a bank, his case may be referred to the
chairman.
(C) ―1‖ above, but has obtained minimum 60% marks at B.Com. As well as CAIIB, his case may be
referred to the managing director.
Based on the above criteria and the information given in each of the following questions, you have
to take the decision in regard of each case. You are not to assume anything. These cases are given
to you as on July 01, 1999.

8. Niranjan‘s father is an industrialist of the city and Niranjan has no problem in keeping any
amount as deposit with the Bank. He has completed B.Sc. with 68% marks and CAIIB with 60%
marks. He is working in a public Sector Bank as an officer in the foreign exchange department
since last two years. His date of birth is 24.4.1974.
1. Refer to Managing Director
2. Refer to General Manager
3. Refer to Chairman
4. Candidate is to be selected
5. Candidate is not to be selected

9. Ashutosh, a commerce graduate with 73% marks, has completed CAIIB part I securing 64%
marks and will be appearing for CAIIB part II examination. His date of birth is 29.6.1975. He
joined a cooperative bank in 1996 and continues to work there. He was posted in the foreign
exchange department in 1997 and is working in the same department at present. He can keep the
required amount of deposit with the bank.
1. Refer to Chairman
2. Refer to General Manager
3. Refer to Managing Director
4. Candidate is to be selected
5. None of these

Analytical Reasoning #3094, Sector 37D, Chandigarh. E-Mail: info@gjtutorial.com Page 79


Institute for All Competitive Exams
10. Manisha is an employee of a private bank in Clerical cadre for past 6 years in its foreign
exchange department. She is B.Com with 70% marks and CAIIB with 60% marks. Her date of birth
is 15.6.1972. She is willing to keep a deposit of any amount up to Rs 50, 000 with the bank.
1. Refer to Chairman
2. Refer to General Manager
3. Refer to Managing Director
4. Candidate is to be selected
5. None of these

11. Rajshekhar, a youth of 26 years age, has completed B.Com. securing 68% marks and is working
in a nationalised bank for past two years in its foreign exchange department in the next month. He
has no problem in depositing Rs 25, 000 with the bank.
1. Refer to General Manager
2. Refer to Managing Director
3. Candidate is to be selected
4. Candidate is not to be selected
5. Data inadequate

12. Abhijit Kaul is a commerce graduate with 72% marks in graduation and 59% marks at CAIIB
examination. He is 28 years old and is working in a bank since 1994 till date. He can keep a deposit
of Rs 25, 000 with the Bank.
1. Refer to Managing Director
2. Refer to General Manager
3. Refer to Chairman
4. Candidate is to be selected
5. Candidate is not to be selected

Directions: Read the following information carefully and answer the questions given below it.
Following are the criteria/conditions for calling candidates for interview for allotting
distributorship of a company.
The candidate must
(a) Be holding a degree in any discipline.
(b) Have secured more than 60 percent marks at S.S.C. and 45 per cent and above marks at degree
level.
(c) Be in the age range of 24 years to 30 years as on 1.5.1998.
(d) Have minimum 2 years experience in the field of marketing or sales.
(e) Be having an income of less than Rs. 1, 20, 000/- per annum.
In case of the applicants, who satisfies all other criteria except:
(i) At (a) above but has 5 years of experience after having obtained 75% and above marks at S.S.C.
and 60% and above marks at H.S.C. be referred to Manager (marketing).
(ii) At (b) above but has more than 60% and above marks at degree level be referred to Sr.
Manager (marketing).
(iii) At (d) above but has held a master‘s degree in marketing management be referred to chief
manager (marketing).
Based on these criteria and information provided below, decide the course of action in each case.
You are not to assume anything. If the data provided is not adequate to decide the given course of
action, your answer will be ―data inadequate‖. The cases are given to you on 1.5.98.

13. Mohan Gandhi holds a degree in Mech. engineering with 70% marks. He has 3 years of
experience in marketing and is earning Rs. 1 lakh per year. He has passed his S.S.C in March 1988
at the age of 16 years, with 55 per cent of marks.
1. Not eligible
2. Data inadequate
Analytical Reasoning #3094, Sector 37D, Chandigarh. E-Mail: info@gjtutorial.com Page 80
Institute for All Competitive Exams
3. Refer to Chief Manager
4. Refer to Sr. Manager
5. Refer to Manager

14. Sangita holds a Master‘s degree in Arts with 82%, 75%, 67% and 58% marks in S.S.C and
H.S.C, B.A. and M.A. respectively. She is in the field of customer sales for the last 3 years and earns
an income of Rs. 7, 000 per month.
1. Call for interview
2. Not eligible
3. Data inadequate
4. Refer to Sr. Manager
5. Refer to Chief Manager

15. Leslie Rebello obtained his S.S.C., with 75 per cent marks and B.Com with 58%. After passing
his B.Com, in 1993 at the age of 20 years, he took up job as Sales Assistant and for last three years
he is working as Asstt. Sales Manager and currently he is having an income of Rs. 9, 500/- per
month. He is doing part-time Master‘s degree in Marketing Management.
1. To be called for interview
2. Not eligible
3. Data inadequate
4. Refer to Sr. Manager
5. Refer to Chief Manager

16. Rajnish Bajaj after obtaining his degree in engineering, is working as Junior Engineer on a
salary of Rs. 6, 000/- per month. He is born on 13th January 1973. He has obtained 85% and 65%
marks at S.S.C and degree respectively. He is currently doing a part-time Master‘s programme in
Marketing Management.
1. Call for interview
2. Data inadequate
3. Not eligible
4. Refer to Chief Manager
5. Refer to Sr. Manager

17. Shobha Galotra has completed recently Master‘s degree in Marketing Management after her
M.Com with 55% marks. Her father is a businessman. She is born on May 15, 1972. Her father‘s
firm has employed her for last two years as Asstt. Manager with monthly salary of Rs. 7, 500/-
1. To be called for interview
2. Data inadequate
3. Refer to Chief Manager
4. Refer to Sr. Manager
5. Not eligible

Directions: Read the following information carefully and answer the questions given below it.
A shipping company chooses merchants navy cadets for employment on both its foreign and
Indian ships. Selection is based on the following criteria. Candidate may be selected for both
foreign and Indian ships based on the following conditions:

a. The candidate has scored at least 75% in class XII.


b. The candidate has minimum height of 5 fts 8 inch.
c. The candidate has a perfect eye sight score of 20/20
d. The candidate is not less than 18 years of age and not more than 20 years as on 1.1.97.
e. The candidate should be able to speak and read Hindi.
(a) If a candidate fulfils all criteria except (c) he is eligible for employment only on foreign ships
Analytical Reasoning #3094, Sector 37D, Chandigarh. E-Mail: info@gjtutorial.com Page 81
Institute for All Competitive Exams
provided he has scored at least 80% in class XII.
(b) If a candidate fulfils all criteria except (d) and he is not older than 21 years as on 1.1.96, he is
eligible for employment only an Indian ship
Mark Answer:
1. If the candidate can be employed on both ships.
2. If the candidate can be employed on a foreign ships.
3. If the candidate can be employed on an Indian ship.
4. If the data is insufficient.
5. If he cannot be employed at all.

18. Deepshikha was born on 15.12.77. She has perfect eye sight and is 5 ft. 11 inch tall, her class XII
percentage was 82, she can speak Hindi fluently but can neither read nor write.

19. Garima's eyesight is 20/20 while her score in class XII 78%. She can speak and read Hindi, she
was born on 26.6.75. This six footer is all set for a merchant navy career.

20. MOHIT KANSAL Singh had cleared class XII exam with 85% and was the state topper in
Hindi this year. This 5ft 9 inch football player had an eyesight score 20/20. He was born in
Chandigarh on 5.11.75.

Answer Key (Conditional Selection)


1: 23344 6: 43454 11: 55431 16: 22255

Analytical Reasoning #3094, Sector 37D, Chandigarh. E-Mail: info@gjtutorial.com Page 82


Institute for All Competitive Exams
Cause Effect Labelling I

Directions: Below are given pairs of events ‗A‘ and ‗B‘. You have to read both the events ‗A‘ and
‗B‘ and decide their nature of relationship. You have to assume that the information given in ‗A‘
and ‗B‘ is true and you will not assume anything beyond the given information in deciding the
answer. Mark answer:
1. If ‗A‘ is the effect and ‗B‘ is its immediate and principal cause;
2. If ‗A‘ is the immediate and principal cause and ‗B‘ is its effect;
3. If ‗A‘ is an effect but ‗B‘ is not its immediate and principal cause; and
4. If ‗B‘ is an effect but ‗A‘ is not its immediate and principal cause.
5. None of these

1.
Event (A): The government has decided to reduce its stake in nationalised banks to 33 per cent
Event (B): The bank employees have gone for one-day strike against the privatisation of public
sector banks

2.
Event (A): The government has decided to roll back the hike in the prices of cooking gas and
kerosene
Event (B): some ministers had resigned in protest against the hike in prices of cooking gas and
other petroleum products

3.
Event (A): The Supreme Court has ordered to close all industrial units in the residential areas in
Delhi
Event (B): A mob of about 1, 500 people went on the rampage and the police has to use teargas to
disperse them and restore normalcy.

4.
Event (A): The brigand Veerappan has released Raj Kumar after 108 days of abduction.
Event (B): The state government ‗A‘ has activated the Special Task Force (STF) to arrest
Veerappan.

5.
Event (A): Cricketer ‗A‘ has denied to talk with media persons.
Event (B): BCCI special commissioner K Madhavan has quizzed cricketer ‗A‘ for nearly six hours
for his role in betting and match-fixing scam.

6.
EVENT (A): The ground has become wet over night.
EVENT (B): It must have rained last night.

7.
EVENT (A): He was not selected for the post of clerk.
EVENT (B): He didn‘t have typing experience and required speed.

8.
EVENT (A): Although the city was under knee-deep water for four days in this monsoon.
EVENT (B): There was no outbreak of Cholera.

9.
Analytical Reasoning #3094, Sector 37D, Chandigarh. E-Mail: info@gjtutorial.com Page 83
Institute for All Competitive Exams
EVENT (A): The working atmosphere in public sector units can only be described as undisciplined
and uncoordinated.
EVENT (B): Govt. has set up a committee to suggest effective measures to improve the efficiency.

10.
EVENT (A): There has been a number of accidents by leaning out of the Railway compartment
EVENT (B): Public notices have been put up by the Railway Department in the compartments to
warn the passengers.

11.
EVENT (A): There has been an severe earthquake in Orissa and the condition of earthquake
victims is pitiable.
EVENT (B): Govt. has released relief funds to help the victims and issued appeal to the public to
donate money for earthquake victims liberally.

12.
EVENT (A): The govt. has increased the procurement price of wheat.
EVENT (B): The price of inputs and labour has increased tremendously during the last year.

13.
EVENT (A): Postal rates have been increased by the govt. to meet the deficit.
EVENT (B): If the rates are not increased, the deficit cannot be met with.

14.
EVENT (A): The function will start at 3 p.m. You are requested to take your seat before 3 p.m.
EVENT (B): The function will start as scheduled.

15.
EVENT (A): The new education policy envisages major modifications in the education system.
EVENT (B): Present education system is inconsistent with national needs.

Answer Key (Cause Effect Labelling I)


1: 21453 6: 11322 11: 23111

Analytical Reasoning #3094, Sector 37D, Chandigarh. E-Mail: info@gjtutorial.com Page 84


Institute for All Competitive Exams
Cause Effect Labelling II

Directions: Below are given pairs of events ‗A‘ and ‗B‘. You have to read both the events ‗A‘ and
‗B‘ and decide their nature of relationship. You have to assume that the information given in ‗A‘
and ‗B‘ is true and you will not assume anything beyond the given information in deciding the
answer. Mark answer:
1. If ‗A‘ is the effect and ‗B‘ is its immediate and principal cause;
2. If ‗A‘ is the immediate and principal cause and ‗B‘ is its effect;
3. If ‗A‘ is an effect but ‗B‘ is not its immediate and principal cause; and
4. If ‗B‘ is an effect but ‗A‘ is not its immediate and principal cause.
5. None of these

1.
EVENT (A): Detergent is used to clean the clothes.
EVENT (B): Detergent helps to dislodge grease and dirt effectively.

2.
EVENT (A): Lock your valuables in a cupboard and call everybody a gentleman.
EVENT (B): Valuables locked in a cupboard will be safe.

3.
EVENT (A): Lack of stimulation in the first four or five years of life can have adverse consequences
on the development of an infant.
EVENT (B): A great part of the development in intelligence occurs in the earliest years of life.

4.
EVENT (A): Govt. has linked the payment of Dearness Allowance with cost index to compensate
the govt. employees.
EVENT (B): There were frequent strikes by the Govt. employees regarding continuous rise in cost
of living and essential commodities.

5.
EVENT (A): There used to be long delay in the preparation results of the competitive examination
as the papers set were of subjective type.
EVENT (B): The govt. has ordered holding of all the competitive examinations in the objective type
pattern.

6.
EVENT (A): Haryana Govt. has introduced punching of attendance card in the offices for all ranks.
EVENT (B): There was a lot of late arrivals and absenteeism in the Govt. offices.

7.
EVENT (A): Most of the influential candidates used to get Govt. service on the basis of forged
educational certificates/ degrees.
EVENT (B): Govt. has made it mandatory that all the original certificates submitted by the Govt.
employee should be got verified from the university concerned.

8.
EVENT (A): Most of the people were buying property in the names of their spouse and not filing
Income tax return to the Govt.
EVENT (B): Filing of the Income Tax return for all incumbents occupying in houses with more
than one thousand square feet has been made compulsory.
Analytical Reasoning #3094, Sector 37D, Chandigarh. E-Mail: info@gjtutorial.com Page 85
Institute for All Competitive Exams
9.
EVENT (A): Govt. has made it mandatory to file Tax Return by all having telephone connection in
their name.
EVENT (B): Most of the people are running business in the names of their wives and having
telephone connection.

10.
EVENT (A): Under the revised govt. rules regarding Income Tax, filing of I.T return has been made
mandatory for all who visits the foreign countries during the current year.
EVENT (B): Rich people were visiting foreign countries as tourists frequently but not submitting
their tax returns to the govt.

11.
EVENT (A): There is a lot of evasion of sale tax in the restaurants and business concerns as cash
memo not issued by the owners.
EVENT (B): The govt. is considering the proposals to install automatic cash accounting machines
to safeguard the concealment of sale and Taxes .

12.
EVENT (A): In army every officer is required to attend refresher courses after a couple of years and
their promotion is based on their ranking in the courses undertaken.
EVENT (B): To equip the officers with the latest techniques of war and use of technology.

13.
EVENT (A): To improve the working knowledge of newly recruited staff in the Govt. service.
EVENT (B): Computer training is necessary to every new entrant in the department.

14.
EVENT (A): Field training is given to every newly appointed Deputy Commissioner so that he is
well conversant with the duties and responsibilities of various subordinate staff working in the
district.
EVENT (B): To equip the newly appointed Deputy Commissioner with the working knowledge of
field staff and responsibilities entrusted to them.

15.
EVENT (A): His children are now serving as officers in Reputed MNCs.
EVENT (B): He has been spending most of his earning on the education of his children.

16.
EVENT (A): He has remained absent from duty continuously for the last so many years without
proper sanction inspite of a number of reminders from the departments.
EVENT (B): His services has been terminated by the department.

17.
EVENT (A): His death has occurred due to failure of vital organs.
EVENT (B): He has been taking liquor daily in large quantity since so many years.

Answer Key (Cause Effect Labelling II)


1: 43214 6: 32433 11: 44354 16: 23

Analytical Reasoning #3094, Sector 37D, Chandigarh. E-Mail: info@gjtutorial.com Page 86


Institute for All Competitive Exams
Inequalities

Directions: In the following questions, the symbols , , , , , are used with the following
meanings:
P  Q means P is not smaller than Q;
P  Q means P is neither greater than nor smaller than Q;
P  Q means P is not greater than Q;
P  Q means P is neither smaller than nor equal to Q; and
P  Q means P is neither greater than nor equal to Q.
Now in each of the following questions, assuming the given statements to be true, find which of the
two conclusion I and II given below them is /are definitely true. Give answer:
1. if only conclusion I is true;
2. if only conclusion II is true;
3. if either I or II is true;
4. if neither I and II is true; and
5. if both I and II are true.

1.
Statements:
MN, HQ, QM
Conclusions:
I. HM
II. QN

2.
Statements:
CB, LS, SC
Conclusions:
I. BS
II. CL

3.
Statements:
IH, EF, IF
Conclusions:
I. E I
II. HE

4.
Statements:
vO, RV, O B
Conclusions:
I. R  B
II. R  B

5.
Statements:
L  U, T L, U W
Conclusion:
I. T  W
II. U  W
Analytical Reasoning #3094, Sector 37D, Chandigarh. E-Mail: info@gjtutorial.com Page 87
Institute for All Competitive Exams

Directions: In the following questions, the symbols #, *, @, $ and = are used with the following
meanings:
S # T means S is greater than T
S * T means S is greater than or equal to T
S @ T means S is equal to T
S $ T means S is lesser than T
S = T means S is lesser than or equal to T
Now in each of the following questions, assuming these statements to be true, find which of the
two conclusions I and II given below them is/are definitely true. Give answer:
1. If only conclusion I is true;
2. If only conclusion II is true;
3. If either conclusion I or conclusion II is true;
4. If neither conclusion I nor conclusion II is true; and
5. If both conclusions I and II are true.

6.
Statements:
N * M, O $ P, M $ P
Conclusions:
I. P # M
II. P * M

7.
Statements:
F # G, G @ H, G # J
Conclusions:
I. G $ F
II. F @ H

8.
Statements:
U # V, W * X, W $ U
Conclusions:
I. W $ V
II. X $ V

9.
Statements:
Q $ R, S # T, R $ T
Conclusions:
I. T # Q
II. S @ R

10.
Statements:
K = L, N $ M, M * K
Conclusions:
I. L @ M
II. M = L

Directions: In the following questions the symbols #, *, @, $ and = are used with the following
meanings:
Analytical Reasoning #3094, Sector 37D, Chandigarh. E-Mail: info@gjtutorial.com Page 88
Institute for All Competitive Exams
A # B means A is greater than B.
A * B means A is greater than or equal to B.
A @ B means A is equal to B.
A $ B means A is lesser than B.
A = B means A is lesser than or equal to B.
Now in each of the following questions, assuming the three statements to be true, find which of the
two conclusions I and II given below them is/are definitely true. Give answer.
1. If only conclusion I is true;
2. If only conclusion II is true;
3. If either conclusions I or conclusions II is true;
4. If neither conclusion I nor conclusions II is true; and
5. If both conclusions I and II are true.

11.
Statements:
F $ G, G = H, H @ J
Conclusions:
I. F @ J
II. H $ F

12.
Statements:
M @ N, O * N, O * P
Conclusions:
I. O @ M
II. P * N

13.
Statements:
B # V, K = C, C = B
Conclusions:
I. V # C
II. B # K

14.
Statements:
K # T, C @ K, T = R
Conclusions:
I. C # R
II. T @ R

15.
Statements:
Y @ M, P * Y, M * B
Conclusions:
I. P @ B
II. P # B

Directions: In the following questions the symbols @, @, =,  $ and $ are used with the following
meaning:
A @ B means A is greater than B.
A @ B means A is either greater than or equal to B.

Analytical Reasoning #3094, Sector 37D, Chandigarh. E-Mail: info@gjtutorial.com Page 89


Institute for All Competitive Exams
A = B means A is equal to B
A $ B means A is smaller than B
A $ B means A is either smaller than or equal to B.
Now in each of the following questions assuming the given statement to be true, find which of the
two conclusions I and / or II given below them is / are definitely true?
Give answer
(1) if only conclusion I is true; give answer
(2) if only conclusion II is true; give answer
(3) if either I or II is true; give answer
(4) if neither I nor II is true and give answer
(5) if both I and II are true.

16.
Statements:
C $ N, N @ R, R @ J
Conclusions:
I. C = J
II. C $ R

17.
Statements:
Z = B, B $ K, T $ Z
Conclusions:
I. T $ K
II. Z @ K

18.
Statements:
H @ D, F $ S, D = S
Conclusions:
I. D @ F
II. D = F

19.
Statements:
K @ T, M $ K, M @ X
Conclusions:
I. T = M
II. K @ X

20.
Statements:
Q @ L, L = C, C $ Z
Conclusions:
IQ@Z
II. L @ Z

Answer Key (Inequalities)


1: 42534 6:11413 11: 44443 16: 41524

Analytical Reasoning #3094, Sector 37D, Chandigarh. E-Mail: info@gjtutorial.com Page 90


Institute for All Competitive Exams

Analytical reasoning
DIRECTIONS for questions 1-6: Refer to the following information and answer the
questions that follow.
A landlord owns a two-storey apartment complex. The complex has four adjacent ground floor
apartments, numbered 111, 112, 113, 114, respectively, and four upstairs apartments numbered 211,
212, 213, 214 such that 211 is directly above 111, 212 is directly above '. '.2, and so on. The landlord
keeps one apartment empty where he stays when he visits town, and rents out six others.
 Nan lives in 114.
 Hal won't live in 214, and Kit won't live next to Liz on the same floor.
 Inga lives directly above Joe.
 Unit 112 is being renovated and can't be rented.

1. If Kit rents 213, then which apartment must Liz rent?


1.211 2.212 3.214 4.111 5.113

2. If Hal rents 111 and Joe rents 113, then which of the following CANNOT happen?
1.211 is empty 2.212isempty 3. 2:-is empty 4. Inga lives in 213 5. Kit lives in

3. Which two tenants live on the same floor?


1. Inga and Liz 2. Liz and Kit 3. Hal and Joe 4.Nan and Kit 5. Nan and Joe

4. Which of the following CANNOT be true if Hal lives next to Nan?


1. Kit lives in 212 2. Kit lives in 213 3. Liz lives in 214 4. Liz lives in 212
5. Inga lives in 211

5. If Inga insists on living next to Hal, then where must Hal live?
1.211 2.212 3.213 4.214 5. On the ground floor

6. If Liz lives in 213, and Nan moves to the apartment that was being renovated, then which
must be true?
1. Hal lives in 113 or 212 2. Kitlivesin211or113 3. Kit lives in 113
4. Inga lives in 211 or 214 5. Hal cannot live in 212

DIRECTIONS for questions: 7- 10: Each question or group of questions is based on a passage
or set of conditions. In answering sonic of the questions, it is best to draw a rough diagram. For
each question, select the best answer choice given.

Five competitors - J, K, L, M and N have completed the first event of a three-event competition,
with K, taking first place, L second place, M third place, J fourth place, and N last place. The
scoring for the three-event competition is as follows:

The winner of the first event receives 5 points; the second-place finisher, 4 points; the third-place
finisher, 3 points; the fourth-place finisher, 2 points; and the last-place finisher, 1 point. The point
values for the second and third event are calculated in the same manner, but the score for the
second event is counted twice in the total point standings for the entire competition.

No ties are possible in the individual events, although there can be ties in the total point standing.
The competitor with the most points after the completion of all three events wins the competition.

Analytical Reasoning #3094, Sector 37D, Chandigarh. E-Mail: info@gjtutorial.com Page 91


Institute for All Competitive Exams
7. If, in the second event, the competitors finish in the same order that they did in the first
event, which of the following could be a possible outcome of the entire competition?
1. J finishes second in total point standings
2. K finishes third in total point standings
3. L finishes third in total point standings
4. M finishes first in total point standings
5. J finishes first in total point standings

8. If only K and M are in a tie for first in total point standings following the completion of the
second event, they must each have how many points?
1. 11 2. 12 3.13 4. 14 5. 18

9. If K finishes third in the second event, the highest possible winning score for the entire
competition is
1.20 2. 16 3. 18 4. 17 5. 19

10. If M and N have equal scores after the completion of the second event, which of the
following CANNOT be true of the outcome of the second event?
1. J finishes higher than M 2. K finished higher than M
3. M finished second and N finished first 4. M finished fourth and N finished second

DIRECTIONS for questions 11-14: Refer to the information given below and answer the
questions that follow.

The countries of Eton, France, Geo, Hartz, Ibiz, Jonia, and Kenya form alliances. Each alliance
contains no fewer than two and no more than four countries.
 Each country enters into one and only one alliance
 Only one of Hartz or Kenya must ally with Ibiz
 Kenya allies with just two other countries
 Eton and Hartz do not ally together

11. If Geo. Ibiz, and Kenya do not ally amongst themselves either altogether or individually,
then Hartz is in an alliance with
1. France 2. Geo 3. Ibiz 4. Jonia 5. Kenya

12. If Kenya allies with Jonia m an "Alliance of Friendship" and just one other alliance is
formed, which other country joins the "Alliance of Friendship"?
1. Eton 2. France 3. Geo 4. Hartz 5 Ibiz

13. If Kenya accepts the invitation from France and Geo to join their single alliance, Eton must
join an alliance.
1. With only Ibiz
2. With only Jonia
3. With Hartz & other countries
4. With Ibiz & two other countries
5. With Jonia and four other countries

14. Which of the following sets of alliances does not violate the conditions specified?
1. Eton/Hartz, Franc/Ibiz/Kenya, Geo/Jonia
Analytical Reasoning #3094, Sector 37D, Chandigarh. E-Mail: info@gjtutorial.com Page 92
Institute for All Competitive Exams
2. Eton/Ibiz, Geo/Hartz, France/lbiz/Kenya
3. Eton/Jonia, Geo/Hartz, France/Ibiz/Kenya
4. Eton/Kenya, Hartz/Jonia, France/Geo/Ibiz
5. Eton/Jonia/Kenya, Hartz/France, Geo/Ibiz

DIRECTIONS for questions 15 - 18: Refer to the information given below and answer the
questions that follow.

Ghosh Balm deposited a certain sum of money in a bank in 1986. The bank calculated interest on
the balance in the account at 10 per cent simple interest, and credited to the account once a year.
At the end of the first year, Ghosh Babu withdrew the entire 20 per cent of the initial amount.
Again at the end of the second year, he withdrew the interest and 50 per cent of the remaining
amount. At the end of the third year, he withdrew the interest and 50 per cent of the remaining
amount. Finally, at the end of the fourth year. Ghosh Babu closed the account and collected the
entire balance of Rs. 11,000.

15. initial amount in rupees, deposited by Ghosh Babii


1.25,000 2.75,000 3. 50,000 4.1,00,000 5.2,00,000

16. The total interest, in rupees, collected by Ghosh Babu was


1.24,000 2.20,000 3.4,000 4.11,000 5.12,000

17. The year, at the end of which, Ghosh Babu withdrew the maximum amount was
1. First 2. Second 3. Third 4. Fourth 5. Can't say

18. The year, at the end of which, Ghosh Babu collected the maximum interest was:
1. First 2. Second 3. Third 4. Fourth 5. Can't say

DIRECTIONS for question 19 & 20: Refer to the information below and answer the questions
that follow.
In a primitive village, only two types of people live who belong to two tribal classes. The first type is
known as class A, while the other is known as class B. In that village, there is no other type of
person except these two. The activities of both types of people are governed by perfectly followed
norms of social behaviour. Each person of the tribe has to obey the norms. They are rigid about
this. As far as marriage is concerned, the following norms are to be followed:
 People of one class cannot marry any other member of their own class, though they can marry
member of the other class.
 After being married, each male member becomes the member of other class to which his wife
belongs, whereas females remain the members of their own class even after being married.
 On birth, the child automatically becomes the member of his or her mother's class.
 If a male member becomes a widower or divorcee, then he again belongs to his group at birth.
 Nobody can marry more than one person at a time according to the social laws.

19. A class A female can have:

1. maternal grandmother from class B.


2. maternal grandfather from class A.
3. paternal grandfather from class A.
4. paternal grandmother from class A
5. mother from class B

20. Which of the following marriages is not permissible by social laws?


Analytical Reasoning #3094, Sector 37D, Chandigarh. E-Mail: info@gjtutorial.com Page 93
Institute for All Competitive Exams
1. Any class B male marries his father's sister.
2. Any class B female marries her mother's brother.
3. Any widower marries his brother's widow.
4. Any class B male marries his grandmother's sister
5. None of these

DIRECTIONS for Questions 21 - 24: Refer to the information given below and answer the
questions that follow.

Sarah invites some friends -A, B, C, D, E, F and G to her home to eat one day. She will serve
breakfast, lunch, and dinner and each friend stays for one or more meals. The following rules will
determine when the friends visit:
 Two consecutive meals may not be served to the same friend.
 Each meal is served to two or more friends.
 Whenever A stays for a meal, B stays for that meal, too.
 F visits for the meal immediately before D's meal.
 F won't come for the same meal as G.

21. Which of the following is an accurate list of visitors who could stay for breakfast?
1. A,C 2.D, F 3. A,F,G 4. C, E, F 5.A,B,F,G

22. Which of the following friends will not come for breakfast?
1. C 2. D 3.E 4. F 5.G

23. If only B, C and D stay for dinner, then which of the following will stay for lunch?
1. E.A 2. E,F 3. E.G 4. F, B 5. G, A

24. If only B, C and G come for breakfast, which of the following will come for dinner?
1. B 2. C 3. E 4. F 5. G

DIRECTIONS for Questions 25 - 30: Refer to the information given below and answer the
questions that follow.

At Bob's birthday party, the children play a game of musical chairs with five chairs placed from left
to right. When the music stops, the children (Zoe, Bob, Chris, Dan, Ed, Fran, Geo, and Hap) try to
sit in the chairs until all the chairs are filled. For each game, the following always happens.
 Each chair holds only one child
 Three children are always left standing.
 The middle chair is occupied by either Ed or Fran.
 Geo and Chris sit next to each other if they are both seated.
 If Geo gets a chair, then Chris won't be left standing.

 Zoe and Hap are never both left standing.


 If Zoe gets seated, she's in the first chair on the left.

25. Which of the following could be the result of one round of musical chairs, seating from left
to right'?
1. Bob, Geo. Fran, Hap, Dan
2. Chris, Geo, Fran, Ed, Hap
3. Geo, Chris. Ed, Fran, Zoe
4. Hap, Bob, Dan, Chris, Geo
Analytical Reasoning #3094, Sector 37D, Chandigarh. E-Mail: info@gjtutorial.com Page 94
Institute for All Competitive Exams
5. Zoe, Ed, Geo, Chris, Dan

26. If Chris sits in the second chair from the left, and Hap is left standing, which of these
children could be sitting next to each other?
1. Zoe and Ed 2. Bob and Chris 3. Chris and Geo 4. Fran and Ed 5. Fran and Geo

27. Suppose Hap sits in the second chair from the right. Who cannot be in the chair on the right
end?
1. Bob 2. Chris 3. Dan 4. Ed 5. Geo

28. If the second chair from the left holds, G, who CANNOT be standing?
1. Bob 2. Ed 3. Fran 4. Hap 5. Zoe

29. If Ed is in the first chair on the left, which of the following must be true?
1. Chris is in the chair second from the right.
2. Fran is in the middle chair.
3. Geo is in the chair second from the right.
4. Geo is in the chair on the right end.
5. Hap is in the chair second from the right.

30. If Dan sits next to Ed, and Fran sits next to Dan, which of the following is correct?
1. Bob must be in a chair on the end.
2. Dan must be in chair second from an end.
3. Ed must be in the middle chair or the chair on the right end.
4. Fran must be in chair on the end.
5. Hap must be in second to the left chair or the right end.

ANALYTICAL REASONING
DIRECTIONS for questions 1 - 4: Read the following information and answer the questions that
follow.

A vending machine has five switches which when operated give Coca-cola, 7-Up, Mirinda, Limca
and Pepsi depending upon which switch is turned on.
The machine is such that each switch supplies two different drinks and each drink is supplied by
two different switches.
If two switches are turned on, the common drinks if any, nullify each other and will not come out
at all.
To get the drink that one wants, one has to turn on the right combination of switches, put in the
money, press the delivery button, and the drink comes out.
(a) 1 and 3, we get, 7-Up and Mirinda.
(b) 2, 4 and 5, we get 7-Up and Mirinda.
(c) 1 and 2, we get Coca-cola and Pepsi.
(d) 1 and 4 we get Limca, Coca-Cola, Mirinda and Pepsi.
(e) 3, 4 and 5, we get Coca-Cola and Pepsi.
(f) 2, 3 and 5, we get Limca, Coca-cola, Mirinda and Pepsi.

Switch 1. 2. 3. 4. 5 do not supply 7-up, Limca, Coca-cola, Mirinda and Pepsi respectively.

1. What drinks are supplied by turning on switch 2 & 3?


1. Mirinda, Limca, 7-Up
2. Pepsi, Limca, 7-Up
3. Coca-cola & Mirinda
Analytical Reasoning #3094, Sector 37D, Chandigarh. E-Mail: info@gjtutorial.com Page 95
Institute for All Competitive Exams
4. Coca-cola, Mirinda, Pepsi & 7-Up
5. Coca - cola, Mirinda & Limca

2. Which switches should be turned on to get just Limca & Coca-cola?


1. 2&3 2. 1,2,3&5 3. 1,3&4 4. 2,3&4 5. 4 & 5

3. Which drink is supplied by turning on switches 2 & 4?


1. Coca-cola, Limca & Mirinda
2. Pepsi, Limca & Mirinda
3. Mirinda & Pepsi vfirinda & Limca
4. Mirinda & Coca - cola

4. Winch drinks are produced by turning on switches 1, 2 & 3?


1. Pepsi, Mirinda, Coca-cola, 7-Up
2. Pepsi, Coca-cola, Limca, 7-Up
3. Mirinda, 7-Up, Limca, Coca-cola
4. Coca-cola & Limca
5. Coca-cola & 7-Up

DIRECTIONS for questions 5 -8: Read the following information and answer the questions
that follow.
Individual members from eight animal species are to be chosen for a special exhibit habitat.
The eight species are P, Q, R. S, T, U, V and W. Because of the way these animals interact, certain
guidelines must be followed.
 Animals that will fight cannot be placed in the habitat together.
 Members of species V will fight with members of species S, T, and U.
 A member of species R will fight with a member of species Q, but only if a member of
species V is present.
 If a member of species W is present, a member of species P will not fight with any animal.
 If a member of species W is not present, a member of species P will fight with members of
species Q&R.
 No fights other than those described above will occur.

5. If V is chosen for the habitat, which of the following CANNOT also be chosen?
1.P 2. Q 3.T 4. R 5.W

6. If two other animals are to be added to a habitat containing a member of species Q and a
member of species V in the habitat, which of the following could be those two animals?
1. Members of species W and P
2. Members of species R and P
3. Members of species S and W
4. Members of species W and R
5. Members of species U and R

7. If two habitats are set up, one containing members of species P, Q, W and V, and the
containing members of species S, U, R and T which animals could be switched one for the
other without provoking any fights?
1. Species W and U 2. Species Q and R
3. Species P and R 4. Species V and S
5. Species W and T

8. If S, P and R are chosen for the habitat, which of the following must also be chosen?
Analytical Reasoning #3094, Sector 37D, Chandigarh. E-Mail: info@gjtutorial.com Page 96
Institute for All Competitive Exams
1. W 2. V 3. U 4.T 5. Q

DIRECTIONS for questions 9 -12: Read the following information and answer the questions
that follow.

A repertory company puts on seven performances a week, one performance every evening. The
plays in the repertory are: Adhe Adhure, which is performed three times a week; Pather Panchali,
which is performed twice a week; Todti Pather, which is performed once a week; and Night of 16th
January, which is performed once a week.

No play should be performed on two consecutive evenings. Todti Pather must be performed either
the evening before or the evening after a performance of Pather Panchali.

9. If Todti Pather is performed on Monday and Pather Panchali on Saturday, all the following
must be true EXCEPT
1. Adhe Adhure is performed on Friday
2. Both performances of Pather Panchali occur on the evening before a performance of
Adhe Adhure
3. Adhe Adhure is performed on Wednesday
4. All three performances of Adhe Adhure occur on the evening either before or after a
performance of Pather Panchali
5. Night of 16th January is performed on Tuesday

10. If Night of 16th January is performed on Monday and Adhe Adhure on Friday, which of the
following MUST be true?
1. Adhe Adhure is performed on Wednesday
2. Todti Pather is performed on Thursday
3. Pather Panchali is performed on Saturday
4. Pather Panchali is performed on Thursday
5. Pather Panchali is performed on Wednesday

11. If Night of 16th January is performed on Sunday and Pather Panchali on Tuesday, which of
the following MUST be true?
1. Adhe Adhure is performed on Saturday
2. Adhe Adhure is performed on Wednesday
3. Pather Panchali is performed on Thursday
4. Todti Pather is performed on Wednesday
5. Pather Panchali is performed on Saturday

12. If Adhe Adhure is performed on Friday, and Pather Panchali is performed on Tuesday,
Night of 16th January could be shown on which of the following pairs of days?
1. Monday or Thursday
2. Monday or Wednesday
3. Wednesday or Thursday
4. Thursday or Saturday
5. Thursday or Sunday

DIRECTIONS for questions 13 -16: Read the following information and answer the questions
that follow.

An assistant principal must assign seven classes - K, L. M, N, O, P and Q --to the seven classrooms
ii the north corridor of her high school. The rooms are numbered 1 through 7, from west to east.
There is a broom closet between Rooms 4 and 5. Only Rooms 1 and 7 have taping facilities.
Analytical Reasoning #3094, Sector 37D, Chandigarh. E-Mail: info@gjtutorial.com Page 97
Institute for All Competitive Exams
 L and M must not be in adjacent rooms.
 N and O must be in adjacent rooms.
 L must be in Room 3.
 Language classes must be in a room with taping facilities.

13. If M and N are both language classes, and if O and L are in adjacent rooms, it must be true
that M is in
1. Room 1
2. Room 7
3. A room adjacent to Q's room
4. A room adjacent to O's room.
5. A room adjacent to K's room.

14. If K. and O are both language classes and if M is not in a room adjacent to the broom closet,
all of the following must be true EXCEPT
1. O is in room 1.
2. N is in room 2.
3. M is in room 6
4. P and L are in adjacent rooms.
5. Q is in a room adjacent to the broom closet.

15. If O is a language class, and if K and Q are in the two rooms adjacent to the broom closet, in
which room could M, be?
1. Room 1
2. Room 2
3. Room 3
4. Room 4
5. Room 5

16. If K and M are in the two rooms adjacent to the broom closet, and if O is in Room 2, which
of the following must be must be true?
1. N is in Room 7.
2. P and M are in adjacent rooms.
3. Q and P are in adjacent rooms.
4. N and K are in adjacent rooms
5. P is in a room with taping facilities.

DIRECTIONS for questions 17 & 18: Read the following information and answer the
questions that follow.

There are 4 persons, Mr. Bhagwan Das, his wife Bhanumati, his son Gopal and his mother Janki.
Of the 4 persons, one is a doctor and another lawyer. Use the following conditions:
(a) If the doctor is younger than the lawyer, then the doctor and the lawyer are not blood
relatives.
(b) If the doctor is a woman, then the doctor and the lawyer are blood relatives.
(c) If the lawyer is a man, then the doctor is a man.

17. Who is the doctor?


1. Bhagwan Das 2. Gopal 3.Janki 4. Bhanumati 5. Can‘t say
18. Who is the lawyer?
1. Bhagwan Das 2. Gopal 3. Janki 4. Bhanumati 5. Can't say

Analytical Reasoning #3094, Sector 37D, Chandigarh. E-Mail: info@gjtutorial.com Page 98


Institute for All Competitive Exams
DIRECTIONS for questions 19 -22: Read the following information and answer the
questions that follow.

Four teams of tennis players, A, B, C, and D, have to be redistributed into three new teams. The
players on team A are L, M, and N. The players on team B are O, P, Q, and R. The players on team
C are S and T. And the players on team D are U, V, and W.

 Each of the three new teams must contain at least one member from three of the original four
teams (teams A, B, C, and D)
 No new team can contain all the members of any of the original teams (teams A, B, C and D).
 V and W cannot be on the same new team.

19. If one new team is made up of L, N, O, and V only, which of the following groups CANNOT
completely represent one of the other new teams?
1. M, P, T
2. P, S, U, W
3. M, R, W
4. Q, R, M, S
5. R, T, W

20. If one new team is made up of N, O, P, Q and U only, which of the following groups
CANNOT completely represent one of the other new teams?
1. R,T,W 2. L,M,T 3.M.S.V 4. M, R, T, W 5. L,R, S,W

21. What is the maximum number of people who can be on the same new team?
1. Five 2. Six 3. Seven 4. Eight 5. Nine

22. If one group is made up of L, O and U only, which of the following could be the complete
roster of another new team?
1. M,N,S,P 2. M, N, P, Q 3. S, V, W, R 4. T, S, V, C 5. T, W, M, R

DIRECTIONS for questions 1 to 4: Read the following information and answer the questions
that follow.

There are five identical looking boxes having different objects in it and every box has a label
indicating their contents. The following is the description of the contents and the label of each box:
Content Label
Two Pins PP
Two Balls BB
Two Clips CC
One Pin and One Clip PC
One Ball and One Clip BC

Somebody has mischievously interchanged these labels in such a way that no box contains the
label correctly explaining its contents

1. If the first box opened contained label PP and the second box opened contained label PC
and out of the combined four items, at least one item was a Ball, which of the following will
be definitely true?
1. Other three items will not contain two Balls.
2. Other three items will not contain any Clip.
3. Other three items will contain at least one Clip.
Analytical Reasoning #3094, Sector 37D, Chandigarh. E-Mail: info@gjtutorial.com Page 99
Institute for All Competitive Exams
4. Other three items will contain at least one Pin.
5. None of these

2. If the box PP contained two Clips, the box CC contained two Pins and the box BB contained
at least one Ball, which of the following will definitely not be true?
1. The box BC contains one Pin and one Clip.
2. The box BB contains one Ball and one Clip
3. The box BC contains two Balls.
4. The box PC contains two Balls.
5. The box PC contains at least one Ball.

3. If the information is available that box PC does not contain either any Pin o .my Clip and
box PP does not contain any Pin and box CC contains one Clip and one Ball, which of the
following will definitely be true if only one of the remaining boxes is opened?
1. It will have one Pin and one Clip.
2. It will have at least one Clip.
3. It will have two Pins.
4. It will have at least one Pin.
5. It will have at least two Clips.

4. If the first box, containing the label BC was opened and it was found that one item is a Ball,
which of the following would be definitely true?
1. The other item may either be a Ball or a Clip.
2. The other box with BB label will contain a Ball and a Clip.
3. The other item will not be a Ball.
4. The other item will be a Pin.
5. The other item will be a Ball.

DIRECTIONS for questions 5 to 7: Refer to the following information and answer the
questions that follow.

Six animals - Hippo, Elephant, Anteater, Cheetah, Sloth bear and Panther are in cages in a single
row, with parks at both ends.
A. The Hippo is next to the park.
B. The Elephant is next to the Anteater.
C. At least one animal separates the Elephant and Cheetah, and the Elephant and the
Sloth bear.
D. The Cheetah and the Sloth bear are next to each other.
E. At least two animals separate the Elephant and the Panther.

5. Which of the following animals can be next to the park?


I. Panther
II. Elephant
III. Cheetah
1. I only 2. II only 3. I and II only 4. I, II and III 5. II & III only

6. If the Hippo is next to the Elephant, which of the following must be true?
1. The Panther is next to a park.
2. The Sloth bear is next to the Anteater.
3. The Panther is next to the Cheetah.
4. The Anteater is next to the Cheetah.
5. The Sloth bear is next to the park.

Analytical Reasoning #3094, Sector 37D, Chandigarh. E-Mail: info@gjtutorial.com Page 100
Institute for All Competitive Exams
7. The Hippo cannot be next to which of the following animals?
1. Anteater only
2. Cheetah only
3. Panther only
4. Cheetah, Sloth bear and Anteater.
5. Elephant only

DIRECTIONS for questions 8 to 11: Refer to the following information and answer the
questions that follow.
Seven commuters - Jack, Kurt, Larry, Mike, Nancy, Oz and Peter - are going on a train that makes
4 halts.
Two commuters get off at every halt, except the last, when the final commuter gets off. The
following constraints exist -
Neither Jack nor Nancy gets off with Kurt.
Larry cannot get off with Mike.
Oz must get off with either Mike or Peter.

8. If Larry gets off with Peter, which of the following must be true?
1. Kurt gets off with Mike.
2. Kurt gets off with Nancy.
3. Jack gets off with Mike.
4. Kurt gets off last.
5. Oz gets off last.

9. Which of the following is an acceptable list of commuters getting off the trains, from the
first halt to the last?
1. Jack and Nancy, Kurt and Peter, Larry and Mike, Oz
2. Jack and Nancy, Kurt and Larry, Oz and Peter, Mike
3. Jack and Kurt, Larry and Peter, Nancy and Oz, Mike
4. Jack and Peter, Larry and Nancy, Kurt and Oz, Mike
5. Jack and Nancy, Larry and Mike, Oz and Peter, Kurt

10. If Larry gets off last, which of the following passengers must get off with Jack?
1. Kurt 2. Mike 3. Oz 4. Peter 5. Nancy

11. If Peter gets off with Nancy, all of the following could be true, except
1. Oz gets off with Mike
2. Kurt gets off with Larry
3. Kurt gets off last
4. Larry gets off last
5. Jack gets off with Larry

DIRECTIONS for questions 12 -16: Read the following information and answer the questions
that follow.

A play director is deciding which and how many of her troupe - A, B, C, D, E, F and G - will act in
her forthcoming play. Using her past experience she frames the following guidelines:

A. If A acts, B must act.


B. If B and C both act, D cannot.
C. If C and D both act, F cannot.
D. If D acts, either E or G must also act.
E. Either E or F must act, but they both cannot act.
Analytical Reasoning #3094, Sector 37D, Chandigarh. E-Mail: info@gjtutorial.com Page 101
Institute for All Competitive Exams
F. E and G cannot both act.

12. If there have to be a minimum of three actors in the play, which of the following c< three?
1.A.B, C 2.B,E, G 3.B,E,F 4. C, D, E 5. A,E,G

13. If D and G both are acting in the play, which of the following must be true?
1. C acts 2. E acts 3. F acts 4. B does not act 5. A acts

14. If D and F are not available for the play, which of the following cannot be true?
1. B acts 2. C acts 3. A does not act 4. E does not act
5. All statements are true

15. If A and C both act, what is the maximum number of actors that the play can have?

1. 3 2. 4 3. 5 4. 6 5. 7

16. If G acts, which of the following combination of three actors can be amongst those who also
act?
1. A.B.E 2. A.B.E 3. A.D.E 4. C, D, F 5. B.D.F

DIRECTIONS for questions 17 to 20: Refer to the following information and answer the questions
that follow.

A new traffic system for a city with seven satellite towns is being planned. The towns are -
Sunshine, Empire, Aloha, Diamond, Bluegrass, Pelican and Wolverine.

The traffic between Empire and Aloha, Diamond and Wolverine, Bluegrass and Pelican will be two
way.. The traffic between the following towns will be one way, from Sunshine to Bluegrass, from
Aloha to Diamond, from Bluegrass to Empire, from Wolverine to Sunshine, from Wolverine to
Aloha,

17. Starting from Empire to reach Bluegrass, at the least how many towns other than Empire
and Bluegrass have to be crossed?
1. 2 2. 3 3. 4 4. 5 5. 6

18. Starting from Aloha to reach Pelican, one has to go through all of the following townships
except
1. Sunshine 2. Empire 3. Diamond 4. Bluegrass 5. Wolverine

19. With the exclusion of the starting and the ending towns, it is possible to have two
alternative routes that have no town in common, if the journey is between
1. Empire and Diamond
2. Empire and Pelican
3. Aloha and Pelican
4. Wolverine and Empire
5. Wolverine and Diamond

20. A traveller going along which of the following routes must touch each of the seven
townships at least once?
1. Sunshine Empire and then to Wolverine
2. Diamond to Aloha and then to Pelican
3. Pelican to Aloha and then to Wolverine
4. Wolverine to Empire and then to Pelican
Analytical Reasoning #3094, Sector 37D, Chandigarh. E-Mail: info@gjtutorial.com Page 102
Institute for All Competitive Exams
5. Empire to Aloha and then to Wolverine

DIRECTIONS for questions 21 to 25: Refer to the following information and answer the
questions that follow.

Alord received a large order for stitching school uniforms from Mayflower school and Little Flower
School. He has two cutters who will cut the fabric; five tailors who will do the stitching and two
assistants to stitch the buttons and two buttonholes. Each of these nine persons will work for
exactly 10 hours a day. Each of the Mayflower uniforms requires 20 minutes for cutting the fabric,
one hour for stitching and 15 minutes for stitching button and buttonholes, whereas the Little
Flower uniform requires after 30 minutes, 1 hour and 30 minutes respectively for these activities.
All the activities can be carried on simultaneously.

21. What is the maximum number of Little Flower uniforms that Alord can complete in a day?
1. 50 2. 20 3. 40 4. 30 5. 60

22. On a particular day, Alord decided to complete 20 Little Flower uniforms. How many
Mayflower uniforms can he complete on the same day?
1. 30 2. 40 3. 20 4. 0 5. 10

23. If Alord decides to complete 30 Little Flower uniforms only and no other on a particular
day, how many total man-hours will be idle?
1. 20 2. 40 3. 5 4. 25 5. 30

24. If he hires one more assistant, what is the maximum number of Mayflower uniforms that he
can complete in a day?
1. 40 2. 50 3. 60 4. 30 5. 45

25. Alord has the option to hire one more employee of any category. Which category should he
hire to get the maximum increase in production capacity, assuming that he needs to stitch
only Mayflower uniforms on that day?
1. Tailor 2. Cutter 3. Assistant 4. Can't say 5. None of these

Analytical Reasoning #3094, Sector 37D, Chandigarh. E-Mail: info@gjtutorial.com Page 103

Vous aimerez peut-être aussi